Rocksolid Light

Welcome to novaBBS (click a section below)

mail  files  register  newsreader  groups  login

Message-ID:  

The system was down for backups from 5am to 10am last Saturday.


tech / sci.physics.relativity / Re: Einstein Paradox: Relativity of Simultaneity

SubjectAuthor
* Einstein Paradox: Relativity of SimultaneityJack Liu
+- Re: Einstein Paradox: Relativity of SimultaneityJack Liu
+- Re: Einstein Paradox: Relativity of SimultaneityRoss Finlayson
+* Re: Einstein Paradox: Relativity of SimultaneityJane
|`* Re: Einstein Paradox: Relativity of SimultaneityJack Liu
| `* Re: Einstein Paradox: Relativity of SimultaneityJane
|  +* Re: Einstein Paradox: Relativity of SimultaneityJack Liu
|  |`* Re: Einstein Paradox: Relativity of SimultaneityJane
|  | `- Re: Einstein Paradox: Relativity of SimultaneityLaurence Clark Crossen
|  `- Re: Einstein Paradox: Relativity of SimultaneityDono.
+* Re: Einstein Paradox: Relativity of SimultaneityJanPB
|+- Re: Einstein Paradox: Relativity of SimultaneityMaciej Wozniak
|+- Re: Einstein Paradox: Relativity of SimultaneityJack Liu
|`* Re: Einstein Paradox: Relativity of SimultaneityVolney
| +* Re: Einstein Paradox: Relativity of SimultaneityJack Liu
| |`* Re: Einstein Paradox: Relativity of SimultaneityVolney
| | +* Re: Einstein Paradox: Relativity of Simultaneitygehan.am...@gmail.com
| | |`* Re: Einstein Paradox: Relativity of SimultaneityVolney
| | | `* Re: Einstein Paradox: Relativity of Simultaneitygehan.am...@gmail.com
| | |  +* Re: Einstein Paradox: Relativity of SimultaneityTom Roberts
| | |  |`* Re: Einstein Paradox: Relativity of Simultaneitygehan.am...@gmail.com
| | |  | +- Re: Einstein Paradox: Relativity of SimultaneityDono.
| | |  | +* Re: Einstein Paradox: Relativity of SimultaneityVolney
| | |  | |+- Re: Einstein Paradox: Relativity of SimultaneityMaciej Wozniak
| | |  | |+- Re: Einstein Paradox: Relativity of SimultaneityJanPB
| | |  | |`- Re: Einstein Paradox: Relativity of SimultaneityTom Roberts
| | |  | `* Re: Einstein Paradox: Relativity of SimultaneityTom Roberts
| | |  |  `* Re: Einstein Paradox: Relativity of Simultaneitygehan.am...@gmail.com
| | |  |   +- Crank Amrit Gehan comes to grips with the fact he's a crankDono.
| | |  |   `* Re: Einstein Paradox: Relativity of SimultaneityTom Roberts
| | |  |    `- Re: Einstein Paradox: Relativity of SimultaneityMaciej Wozniak
| | |  `- Re: Einstein Paradox: Relativity of SimultaneityVolney
| | `- Re: Einstein Paradox: Relativity of SimultaneityEvenezer Nigro
| +* Re: Einstein Paradox: Relativity of SimultaneityProkaryotic Capase Homolog
| |+* Re: Einstein Paradox: Relativity of SimultaneityJack Liu
| ||`* Re: Einstein Paradox: Relativity of SimultaneityProkaryotic Capase Homolog
| || `* Re: Einstein Paradox: Relativity of SimultaneityJack Liu
| ||  +* Re: Einstein Paradox: Relativity of SimultaneityProkaryotic Capase Homolog
| ||  |`- Re: Einstein Paradox: Relativity of SimultaneityMaciej Wozniak
| ||  +- Re: Einstein Paradox: Relativity of SimultaneityLaurence Clark Crossen
| ||  `* Re: Einstein Paradox: Relativity of SimultaneityRichard Hachel
| ||   `* Re: Einstein Paradox: Relativity of SimultaneityJack Liu
| ||    `* Re: Einstein Paradox: Relativity of SimultaneityRichard Hachel
| ||     `* Re: Einstein Paradox: Relativity of SimultaneityJack Liu
| ||      `* Re: Einstein Paradox: Relativity of SimultaneityRichard Hachel
| ||       `- Re: Einstein Paradox: Relativity of SimultaneityJack Liu
| |`- Re: Einstein Paradox: Relativity of SimultaneityProkaryotic Capase Homolog
| +- Re: Einstein Paradox: Relativity of SimultaneityMaciej Wozniak
| +- Re: Einstein Paradox: Relativity of SimultaneityEvenezer Nigro
| `* Re: Einstein Paradox: Relativity of Simultaneitygehan.am...@gmail.com
|  `* Re: Einstein Paradox: Relativity of SimultaneityVolney
|   +* Re: Einstein Paradox: Relativity of Simultaneitygehan.am...@gmail.com
|   |+- Re: Einstein Paradox: Relativity of SimultaneityJack Liu
|   |`* Re: Einstein Paradox: Relativity of SimultaneityVolney
|   | +* Re: Einstein Paradox: Relativity of SimultaneityJack Liu
|   | |`* Re: Einstein Paradox: Relativity of SimultaneityPython
|   | | `* Re: Einstein Paradox: Relativity of SimultaneityMaciej Wozniak
|   | |  `- Re: Einstein Paradox: Relativity of Simultaneitymitchr...@gmail.com
|   | `* Re: Einstein Paradox: Relativity of Simultaneitygehan.am...@gmail.com
|   |  `* Re: Einstein Paradox: Relativity of SimultaneityVolney
|   |   `- Re: Einstein Paradox: Relativity of SimultaneityMaciej Wozniak
|   `- Re: Einstein Paradox: Relativity of SimultaneityMaciej Wozniak
+* Re: Einstein Paradox: Relativity of Simultaneitymitchr...@gmail.com
|`- Re: Einstein Paradox: Relativity of SimultaneityEvenezer Nigro
`* Re: Einstein Paradox: Relativity of SimultaneitySylvia Else
 +* Re: Einstein Paradox: Relativity of SimultaneityProkaryotic Capase Homolog
 |+- Re: Einstein Paradox: Relativity of SimultaneityProkaryotic Capase Homolog
 |+* Re: Einstein Paradox: Relativity of SimultaneityJack Liu
 ||`- Re: Einstein Paradox: Relativity of SimultaneityPaparios
 |+* Re: Einstein Paradox: Relativity of Simultaneitygehan.am...@gmail.com
 ||`* Re: Einstein Paradox: Relativity of SimultaneityProkaryotic Capase Homolog
 || +- Re: Einstein Paradox: Relativity of SimultaneityMaciej Wozniak
 || +* Re: Einstein Paradox: Relativity of Simultaneitygehan.am...@gmail.com
 || |`* Re: Einstein Paradox: Relativity of SimultaneityProkaryotic Capase Homolog
 || | +- Re: Einstein Paradox: Relativity of SimultaneityProkaryotic Capase Homolog
 || | `* Re: Einstein Paradox: Relativity of Simultaneitygehan.am...@gmail.com
 || |  +* Re: Einstein Paradox: Relativity of SimultaneityMaciej Wozniak
 || |  |`* Re: Einstein Paradox: Relativity of Simultaneitygehan.am...@gmail.com
 || |  | `- Re: Einstein Paradox: Relativity of SimultaneityMaciej Wozniak
 || |  `- Re: Einstein Paradox: Relativity of SimultaneityProkaryotic Capase Homolog
 || `- Re: Einstein Paradox: Relativity of SimultaneityRichD
 |`* Re: Einstein Paradox: Relativity of SimultaneityRichD
 | `* Re: Einstein Paradox: Relativity of SimultaneityProkaryotic Capase Homolog
 |  `* Re: Einstein Paradox: Relativity of SimultaneityProkaryotic Capase Homolog
 |   +- Re: Einstein Paradox: Relativity of SimultaneityMaciej Wozniak
 |   `- Re: Einstein Paradox: Relativity of SimultaneityRichD
 +- Re: Einstein Paradox: Relativity of SimultaneityJack Liu
 +* Re: Einstein Paradox: Relativity of SimultaneityJack Liu
 |`* Re: Einstein Paradox: Relativity of SimultaneitySylvia Else
 | `* Re: Einstein Paradox: Relativity of SimultaneityJack Liu
 |  `* Re: Einstein Paradox: Relativity of SimultaneityLaurence Clark Crossen
 |   `- Re: Einstein Paradox: Relativity of SimultaneityJack Liu
 +- Re: Einstein Paradox: Relativity of SimultaneityEvenezer Nigro
 `- Re: Einstein Paradox: Relativity of SimultaneityEvenezer Nigro

Pages:1234
Re: Einstein Paradox: Relativity of Simultaneity

<u2hgks$2j7m2$4@dont-email.me>

  copy mid

https://www.novabbs.com/tech/article-flat.php?id=113851&group=sci.physics.relativity#113851

  copy link   Newsgroups: sci.physics.relativity sci.physics sci.math
Path: i2pn2.org!i2pn.org!eternal-september.org!news.eternal-september.org!.POSTED!not-for-mail
From: veo...@rnigeern.ne (Evenezer Nigro)
Newsgroups: sci.physics.relativity,sci.physics,sci.math
Subject: Re: Einstein Paradox: Relativity of Simultaneity
Date: Fri, 28 Apr 2023 22:17:00 -0000 (UTC)
Organization: A noiseless patient Spider
Lines: 25
Message-ID: <u2hgks$2j7m2$4@dont-email.me>
References: <887d51c1-c60c-4d24-993e-7941d85c4311n@googlegroups.com>
<d2db6956-eba1-4a39-9a09-2a5fbaee5fcdn@googlegroups.com>
<u2gpgn$2gg8j$1@dont-email.me>
MIME-Version: 1.0
Content-Type: text/plain; charset=UTF-8
Content-Transfer-Encoding: 8bit
Injection-Date: Fri, 28 Apr 2023 22:17:00 -0000 (UTC)
Injection-Info: dont-email.me; posting-host="4a60a02369a758ad513d9b378447ddf6";
logging-data="2727618"; mail-complaints-to="abuse@eternal-september.org"; posting-account="U2FsdGVkX1+8ARu3ODK7R2iWCj32NOS6"
User-Agent: Chrome/82.0.3496.87 Mobile Safari/632.24
Cancel-Lock: sha1:iQaPZ+XjYimwmx/A1TNGRc19lJQ=
Face: iVBORw0KGgoAAAANSUhEUgAAADAAAAAwBAMAAAClLOS0AAAAHlBMVEXq09M5hMsY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X-Face: #TSoad.,*\Pt@O|Rpv$5n7{R&d)3Y_dq'vjGWi]>vqcE^1-{.#b>6$CM3wJeYl,{
SvE,`f-M5r(VCdaV~YHU8nG'9Z#AbJul\o<5m9~9jd.:dY|]h_?,A"72Dv_5f,fR2[G5P{#
L~bIG{.S"xEG@@Ae9}VVMvaO0q_~Vwsm==c-?nQDE(AzQ~bK,:CK>Zp6I~#e1H>5kXentB[
F15s3A&lhy8EIeoluw$V>"*=O=nl8s{*l6L,Y(8}W~LWN"bJdz7RUH`mSb=(i3oJCZuglfE
1&I^vU74paf6'!##k~?QNKohE0~F-3yEmS'UU4`+?:
 by: Evenezer Nigro - Fri, 28 Apr 2023 22:17 UTC

Volney wrote:

> On 4/28/2023 3:43 AM, JanPB wrote:
>> Simultaneity is not essential for SR, it's merely a convenience for
>> both derivation and use. But not for the theory as such.
>
> Also, relativity of simultaneity doesn't depend on length
> contraction/time dilation in SR in all cases.
>
> Consider Einstein's train of length L, but is STATIONARY (stopped).
> Observer A is on the embankment at the midpoint of the train. Observer B
> is on the embankment at the front of the train. Observer C is on the
> embankment at the rear.

SHOCK! Woman speaks on HORRORS of Russian Occupation of LPR!
https://b%69%74%63%68ute.com/video/VYA6glNF2CX5

Azerbaijani President Owns Western BBC Presstitute
https://b%69%74%63%68ute.com/video/BjW06pxZiars

the *_climate_change_* "experts" doesn't know the percentage of CO2 in the
air they breath. Lol. I fuck your ass. They run at speed of *_"science"_*.

Farmer asks a simple question from the climate experts
https://b%69%74%63%68ute.com/video/dpFz6YcPTbLc

Re: Einstein Paradox: Relativity of Simultaneity

<35fc3de8-b325-4be7-804d-1e3364fe1dd7n@googlegroups.com>

  copy mid

https://www.novabbs.com/tech/article-flat.php?id=113870&group=sci.physics.relativity#113870

  copy link   Newsgroups: sci.physics.relativity
X-Received: by 2002:ac8:5c53:0:b0:3eb:14c0:b41b with SMTP id j19-20020ac85c53000000b003eb14c0b41bmr2560222qtj.5.1682728698195;
Fri, 28 Apr 2023 17:38:18 -0700 (PDT)
X-Received: by 2002:a05:622a:1822:b0:3f0:a887:7d45 with SMTP id
t34-20020a05622a182200b003f0a8877d45mr2498308qtc.13.1682728697948; Fri, 28
Apr 2023 17:38:17 -0700 (PDT)
Path: i2pn2.org!i2pn.org!usenet.goja.nl.eu.org!3.eu.feeder.erje.net!2.eu.feeder.erje.net!feeder.erje.net!proxad.net!feeder1-2.proxad.net!209.85.160.216.MISMATCH!news-out.google.com!nntp.google.com!postnews.google.com!google-groups.googlegroups.com!not-for-mail
Newsgroups: sci.physics.relativity
Date: Fri, 28 Apr 2023 17:38:17 -0700 (PDT)
In-Reply-To: <_5CJAa4qjAloBDuFOEVK0NIA2m0@jntp>
Injection-Info: google-groups.googlegroups.com; posting-host=98.197.62.158; posting-account=8fhuCAkAAADWKcWwPpTEbux5C4yu40ip
NNTP-Posting-Host: 98.197.62.158
References: <887d51c1-c60c-4d24-993e-7941d85c4311n@googlegroups.com>
<d2db6956-eba1-4a39-9a09-2a5fbaee5fcdn@googlegroups.com> <u2gpgn$2gg8j$1@dont-email.me>
<5cdada26-5b79-4620-8ac1-7602bf5442c5n@googlegroups.com> <c87d1b2f-9e7c-4411-81cd-20a2f271c8c4n@googlegroups.com>
<0747548f-bc47-46d7-b0c2-68d09c3a7e9an@googlegroups.com> <e51ef3fa-5e52-48e6-bf4a-3f8499f876adn@googlegroups.com>
<_5CJAa4qjAloBDuFOEVK0NIA2m0@jntp>
User-Agent: G2/1.0
MIME-Version: 1.0
Message-ID: <35fc3de8-b325-4be7-804d-1e3364fe1dd7n@googlegroups.com>
Subject: Re: Einstein Paradox: Relativity of Simultaneity
From: liu...@gmail.com (Jack Liu)
Injection-Date: Sat, 29 Apr 2023 00:38:18 +0000
Content-Type: text/plain; charset="UTF-8"
Content-Transfer-Encoding: quoted-printable
 by: Jack Liu - Sat, 29 Apr 2023 00:38 UTC

On Friday, April 28, 2023 at 5:10:35 PM UTC-5, Richard Hachel wrote:
> Le 28/04/2023 à 19:33, Jack Liu a écrit :
>
> > Jack
>
> I found this in your book.
>
> What does it mean?
>
> <http://news2.nemoweb.net/jntp?_5CJAa4qjAloBDuFOEVK0NIA2m0@jntp/Data.Media:1>
>
> <http://news2.nemoweb.net/?DataID=_5CJAa4qjAloBDuFOEVK0NIA2m0@jntp>
>
> R.H.

Dear R. H.

Thank you for reading my book.

That is Lorentz Factor I derive for inbound motion Lorentz Transformation.

Einstein consider only outbound moving body and derive a Lorentz factor > 0, which indicate time dilation. That is only half of the whole picture.
I apply same logic of Einstein to apply to inbound moving body, and derive to above new Lorentz factor which is <1, which indicate time contraction.

I try to demonstrate that, once SP continue to develop another half, it will contradict to itself.

Jack

Re: Einstein Paradox: Relativity of Simultaneity

<u2hrtq$2lh6h$1@dont-email.me>

  copy mid

https://www.novabbs.com/tech/article-flat.php?id=113875&group=sci.physics.relativity#113875

  copy link   Newsgroups: sci.physics.relativity
Path: i2pn2.org!i2pn.org!eternal-september.org!news.eternal-september.org!.POSTED!not-for-mail
From: vol...@invalid.invalid (Volney)
Newsgroups: sci.physics.relativity
Subject: Re: Einstein Paradox: Relativity of Simultaneity
Date: Fri, 28 Apr 2023 21:29:35 -0400
Organization: A noiseless patient Spider
Lines: 47
Message-ID: <u2hrtq$2lh6h$1@dont-email.me>
References: <887d51c1-c60c-4d24-993e-7941d85c4311n@googlegroups.com>
<d2db6956-eba1-4a39-9a09-2a5fbaee5fcdn@googlegroups.com>
<u2gpgn$2gg8j$1@dont-email.me>
<3415ec27-f4bd-45dd-8b3c-10711c462eccn@googlegroups.com>
MIME-Version: 1.0
Content-Type: text/plain; charset=UTF-8; format=flowed
Content-Transfer-Encoding: 8bit
Injection-Date: Sat, 29 Apr 2023 01:29:31 -0000 (UTC)
Injection-Info: dont-email.me; posting-host="5abf68a36468999275cfec4cf545e99d";
logging-data="2802897"; mail-complaints-to="abuse@eternal-september.org"; posting-account="U2FsdGVkX18urWb0cqLvcY8IRRqrk2bw"
User-Agent: Mozilla/5.0 (Windows NT 6.1; Win64; x64; rv:102.0) Gecko/20100101
Thunderbird/102.10.0
Cancel-Lock: sha1:sGabR0yC5O0HuliFsKC51TVIizg=
Content-Language: en-US
In-Reply-To: <3415ec27-f4bd-45dd-8b3c-10711c462eccn@googlegroups.com>
 by: Volney - Sat, 29 Apr 2023 01:29 UTC

On 4/28/2023 12:15 PM, Jack Liu wrote:
> On Friday, April 28, 2023 at 10:42:18 AM UTC-5, Volney wrote:
>> On 4/28/2023 3:43 AM, JanPB wrote:
>>> On Tuesday, April 25, 2023 at 2:23:08 PM UTC-7, Jack Liu wrote:
>>>> Einstein's theory of simultaneity is nothing less than a paradox. I would name it Einstein's Paradox. This should be the biggest paradox in physics in the 20th century. (chapter 9 in "absolute time")
>>>
>>> Simultaneity is not essential for SR, it's merely a convenience for
>>> both derivation and use. But not for the theory as such.
>> Also, relativity of simultaneity doesn't depend on length
>> contraction/time dilation in SR in all cases.
>>
>> Consider Einstein's train of length L, but is STATIONARY (stopped).
>> Observer A is on the embankment at the midpoint of the train. Observer B
>> is on the embankment at the front of the train. Observer C is on the
>> embankment at the rear.
>>
>> Observer A sees simultaneous lightning strikes at the front and rear of
>> the train. A says the strikes are simultaneous, as the light from each
>> reaches her at the same time. A observes the strikes L/2c after they
>> actually happen.
>>
>> What does B see? B sees the strike on the front and time L/c later sees
>> the strike on the rear.
>>
>> What does C see? C sees the strike on the rear and time L/c later sees
>> the strike on the front.
>>
>> No SR, no GR, no length contraction or time dilation, nothing but a
>> finite speed of light. But the three observers all disagree on the
>> simultaneity of the lightning strikes.
>
> To Volney
>
> You are right A, B, C has different about the timing of lightning, about the simultaneity of two event. However Einstein just believes in absolute time by claiming two lightning strike the two end of train SAME TIME at his premise.

No, Einstein stated Observer M at the midpoint on the embankment SEES
the flashes at the same time. That simply means the light from the two
flashes enters his eyes at the same time. Because of this M concludes
the flashes are simultaneous.
>
> It is so ridiculous to think Relativity revolution has overthrow the absolute time while Einstein himself trust absolute time.

No, Einstein never uses absolute time. It's called the Theory of
RELATIVITY, in part because time is relative, not absolute.

If you believe the 1905 paper or any of Einstein's gedankens use
absolute time, you simply don't understand them.

Re: Einstein Paradox: Relativity of Simultaneity

<0ec41357-3ba8-4f93-9979-d8d0036fa660n@googlegroups.com>

  copy mid

https://www.novabbs.com/tech/article-flat.php?id=113890&group=sci.physics.relativity#113890

  copy link   Newsgroups: sci.physics.relativity
X-Received: by 2002:a05:622a:109:b0:3f0:a7ba:74a with SMTP id u9-20020a05622a010900b003f0a7ba074amr2440798qtw.4.1682736971083;
Fri, 28 Apr 2023 19:56:11 -0700 (PDT)
X-Received: by 2002:a05:620a:127a:b0:74e:32ca:d7f0 with SMTP id
b26-20020a05620a127a00b0074e32cad7f0mr1300786qkl.11.1682736970885; Fri, 28
Apr 2023 19:56:10 -0700 (PDT)
Path: i2pn2.org!i2pn.org!weretis.net!feeder6.news.weretis.net!nntp.club.cc.cmu.edu!45.76.7.193.MISMATCH!3.us.feeder.erje.net!feeder.erje.net!border-1.nntp.ord.giganews.com!nntp.giganews.com!news-out.google.com!nntp.google.com!postnews.google.com!google-groups.googlegroups.com!not-for-mail
Newsgroups: sci.physics.relativity
Date: Fri, 28 Apr 2023 19:56:10 -0700 (PDT)
In-Reply-To: <u2gpgn$2gg8j$1@dont-email.me>
Injection-Info: google-groups.googlegroups.com; posting-host=185.215.32.55; posting-account=sVBCDQoAAAADe-Ogi2R38m91EmLrcIgt
NNTP-Posting-Host: 185.215.32.55
References: <887d51c1-c60c-4d24-993e-7941d85c4311n@googlegroups.com>
<d2db6956-eba1-4a39-9a09-2a5fbaee5fcdn@googlegroups.com> <u2gpgn$2gg8j$1@dont-email.me>
User-Agent: G2/1.0
MIME-Version: 1.0
Message-ID: <0ec41357-3ba8-4f93-9979-d8d0036fa660n@googlegroups.com>
Subject: Re: Einstein Paradox: Relativity of Simultaneity
From: gehan.am...@gmail.com (gehan.am...@gmail.com)
Injection-Date: Sat, 29 Apr 2023 02:56:11 +0000
Content-Type: text/plain; charset="UTF-8"
Content-Transfer-Encoding: quoted-printable
Lines: 97
 by: gehan.am...@gmail.co - Sat, 29 Apr 2023 02:56 UTC

On Friday, April 28, 2023 at 8:42:18 PM UTC+5, Volney wrote:
> On 4/28/2023 3:43 AM, JanPB wrote:
> > On Tuesday, April 25, 2023 at 2:23:08 PM UTC-7, Jack Liu wrote:
> >> Einstein's theory of simultaneity is nothing less than a paradox. I would name it Einstein's Paradox. This should be the biggest paradox in physics in the 20th century. (chapter 9 in "absolute time")
> >
> > Simultaneity is not essential for SR, it's merely a convenience for
> > both derivation and use. But not for the theory as such.
> Also, relativity of simultaneity doesn't depend on length
> contraction/time dilation in SR in all cases.
>
> Consider Einstein's train of length L, but is STATIONARY (stopped).
> Observer A is on the embankment at the midpoint of the train. Observer B
> is on the embankment at the front of the train. Observer C is on the
> embankment at the rear.
>
> Observer A sees simultaneous lightning strikes at the front and rear of
> the train. A says the strikes are simultaneous, as the light from each
> reaches her at the same time. A observes the strikes L/2c after they
> actually happen.
>
> What does B see? B sees the strike on the front and time L/c later sees
> the strike on the rear.
>
> What does C see? C sees the strike on the rear and time L/c later sees
> the strike on the front.
>
> No SR, no GR, no length contraction or time dilation, nothing but a
> finite speed of light. But the three observers all disagree on the
> simultaneity of the lightning strikes.

This is because the person closer to the lightning strikes sees it first.

The unstated assumption is that the person on the train does not know he is moving.

This is why I call it a thoughtless experiment.

What about another thought experiment?

What if the lightning strikes strike the train and the tracks at each end respectively?

Which sources are 'moving'?

No one will answer this question satisfactorily. Except maybe ChatGPT.

######
Q:There is a long train with a person at the midpoint, and a person on the tracks next to the train. At the time that the person in the middle of the train is opposite the person on the tracks, lightning strikes the ends of the train and the tracks at the same time. Now what will the observer on the train see, call him A, and what will the observer on the tracks see, call him B? What is the difference in the velocity of light from each of the four strikes in B's frame of reference, and A's frame of reference?

A:
According to the principle of relativity, the laws of physics are the same in all inertial frames of reference, so the observers on the train and on the tracks will both observe the same events, but they may describe them differently.

In the reference frame of observer A on the train, they will see the lightning strikes at the ends of the train occur simultaneously since the speed of light is constant for all observers. The light from the lightning strikes will reach observer A at the same time, regardless of whether they are at the front or the back of the train.

In the reference frame of observer B on the tracks, they will also see the lightning strikes at the ends of the train occur simultaneously since the speed of light is constant for all observers. However, because the train is moving, observer B will see the person in the middle of the train moving relative to them, and the distance that the light has to travel to reach the observer from each lightning strike is different. As a result, observer B will measure a difference in the time it takes for the light to reach them from each of the four strikes.

Specifically, in observer B's frame of reference, the light from the lightning strike at the back of the train has to travel a shorter distance to reach them since the train is moving towards them. Conversely, the light from the lightning strike at the front of the train has to travel a longer distance since the train is moving away from them. Therefore, observer B will measure a time difference between the arrival of the light from the two strikes, which is proportional to the distance traveled by the light and the relative velocity between the train and observer B.

However, since the speed of light is constant for all observers, both A and B will measure the same value for the speed of light in their respective frames of reference, and there will be no difference in the velocity of light from each of the four strikes in either A's or B's frame of reference.

######

Re: Einstein Paradox: Relativity of Simultaneity

<b135a95f-a251-47eb-966c-7acf54465952n@googlegroups.com>

  copy mid

https://www.novabbs.com/tech/article-flat.php?id=113891&group=sci.physics.relativity#113891

  copy link   Newsgroups: sci.physics.relativity
X-Received: by 2002:ac8:5c53:0:b0:3e1:6129:f094 with SMTP id j19-20020ac85c53000000b003e16129f094mr2583560qtj.7.1682737201702;
Fri, 28 Apr 2023 20:00:01 -0700 (PDT)
X-Received: by 2002:a05:622a:1885:b0:3e6:3806:70e3 with SMTP id
v5-20020a05622a188500b003e6380670e3mr2548774qtc.8.1682737201388; Fri, 28 Apr
2023 20:00:01 -0700 (PDT)
Path: i2pn2.org!i2pn.org!weretis.net!feeder6.news.weretis.net!news.misty.com!border-2.nntp.ord.giganews.com!nntp.giganews.com!news-out.google.com!nntp.google.com!postnews.google.com!google-groups.googlegroups.com!not-for-mail
Newsgroups: sci.physics.relativity
Date: Fri, 28 Apr 2023 20:00:01 -0700 (PDT)
In-Reply-To: <u2hrtq$2lh6h$1@dont-email.me>
Injection-Info: google-groups.googlegroups.com; posting-host=185.215.32.55; posting-account=sVBCDQoAAAADe-Ogi2R38m91EmLrcIgt
NNTP-Posting-Host: 185.215.32.55
References: <887d51c1-c60c-4d24-993e-7941d85c4311n@googlegroups.com>
<d2db6956-eba1-4a39-9a09-2a5fbaee5fcdn@googlegroups.com> <u2gpgn$2gg8j$1@dont-email.me>
<3415ec27-f4bd-45dd-8b3c-10711c462eccn@googlegroups.com> <u2hrtq$2lh6h$1@dont-email.me>
User-Agent: G2/1.0
MIME-Version: 1.0
Message-ID: <b135a95f-a251-47eb-966c-7acf54465952n@googlegroups.com>
Subject: Re: Einstein Paradox: Relativity of Simultaneity
From: gehan.am...@gmail.com (gehan.am...@gmail.com)
Injection-Date: Sat, 29 Apr 2023 03:00:01 +0000
Content-Type: text/plain; charset="UTF-8"
Content-Transfer-Encoding: quoted-printable
Lines: 70
 by: gehan.am...@gmail.co - Sat, 29 Apr 2023 03:00 UTC

On Saturday, April 29, 2023 at 6:29:33 AM UTC+5, Volney wrote:
> On 4/28/2023 12:15 PM, Jack Liu wrote:
> > On Friday, April 28, 2023 at 10:42:18 AM UTC-5, Volney wrote:
> >> On 4/28/2023 3:43 AM, JanPB wrote:
> >>> On Tuesday, April 25, 2023 at 2:23:08 PM UTC-7, Jack Liu wrote:
> >>>> Einstein's theory of simultaneity is nothing less than a paradox. I would name it Einstein's Paradox. This should be the biggest paradox in physics in the 20th century. (chapter 9 in "absolute time")
> >>>
> >>> Simultaneity is not essential for SR, it's merely a convenience for
> >>> both derivation and use. But not for the theory as such.
> >> Also, relativity of simultaneity doesn't depend on length
> >> contraction/time dilation in SR in all cases.
> >>
> >> Consider Einstein's train of length L, but is STATIONARY (stopped).
> >> Observer A is on the embankment at the midpoint of the train. Observer B
> >> is on the embankment at the front of the train. Observer C is on the
> >> embankment at the rear.
> >>
> >> Observer A sees simultaneous lightning strikes at the front and rear of
> >> the train. A says the strikes are simultaneous, as the light from each
> >> reaches her at the same time. A observes the strikes L/2c after they
> >> actually happen.
> >>
> >> What does B see? B sees the strike on the front and time L/c later sees
> >> the strike on the rear.
> >>
> >> What does C see? C sees the strike on the rear and time L/c later sees
> >> the strike on the front.
> >>
> >> No SR, no GR, no length contraction or time dilation, nothing but a
> >> finite speed of light. But the three observers all disagree on the
> >> simultaneity of the lightning strikes.
> >
> > To Volney
> >
> > You are right A, B, C has different about the timing of lightning, about the simultaneity of two event. However Einstein just believes in absolute time by claiming two lightning strike the two end of train SAME TIME at his premise.
> No, Einstein stated Observer M at the midpoint on the embankment SEES
> the flashes at the same time. That simply means the light from the two
> flashes enters his eyes at the same time. Because of this M concludes
> the flashes are simultaneous.
> >
> > It is so ridiculous to think Relativity revolution has overthrow the absolute time while Einstein himself trust absolute time.
> No, Einstein never uses absolute time. It's called the Theory of
> RELATIVITY, in part because time is relative, not absolute.
>
> If you believe the 1905 paper or any of Einstein's gedankens use
> absolute time, you simply don't understand them.
Einstein did not use relativity in his relativity paper. That would be circular.
He used Newtonian mechanics plus the second postulate to draw some conclusions.

Time stands still for the surfer and he can never travel faster than the wave.

But he can hit a surfboard at 2c

Re: Einstein Paradox: Relativity of Simultaneity

<kb3plpFa9hkU1@mid.individual.net>

  copy mid

https://www.novabbs.com/tech/article-flat.php?id=113912&group=sci.physics.relativity#113912

  copy link   Newsgroups: sci.physics.relativity
Path: i2pn2.org!i2pn.org!weretis.net!feeder8.news.weretis.net!news.imp.ch!fu-berlin.de!uni-berlin.de!individual.net!not-for-mail
From: syl...@email.invalid (Sylvia Else)
Newsgroups: sci.physics.relativity
Subject: Re: Einstein Paradox: Relativity of Simultaneity
Date: Sat, 29 Apr 2023 15:52:57 +1000
Lines: 19
Message-ID: <kb3plpFa9hkU1@mid.individual.net>
References: <887d51c1-c60c-4d24-993e-7941d85c4311n@googlegroups.com>
Mime-Version: 1.0
Content-Type: text/plain; charset=UTF-8; format=flowed
Content-Transfer-Encoding: 7bit
X-Trace: individual.net w+tK8agNwYlS9XsZi7wFOgjBRgy7rh6fFYQKdHOynrpm/FLI7D
Cancel-Lock: sha1:5leSySS0za1jCzYSg0FDnWUMPEY=
User-Agent: Mozilla/5.0 (Windows NT 10.0; Win64; x64; rv:102.0) Gecko/20100101
Thunderbird/102.10.1
Content-Language: en-GB
In-Reply-To: <887d51c1-c60c-4d24-993e-7941d85c4311n@googlegroups.com>
 by: Sylvia Else - Sat, 29 Apr 2023 05:52 UTC

On 26-Apr-23 7:23 am, Jack Liu wrote:
> Einstein's theory of simultaneity is nothing less than a paradox. I would name it Einstein's Paradox. This should be the biggest paradox in physics in the 20th century. (chapter 9 in "absolute time")
>
> How did Einstein analyze simultaneity relativity? He first assumed that two lightning bolts hit the rails at both ends of the railway "simultaneously", and then he analyzed whether the lightning hit the rails "simultaneously" in the perspective of two observers located at that very moment in the center of the rail.
> The focus is not on whether two observers experience different "simultaneity". The point is that Einstein's reasoning was inconsistent. His conclusion directly denies his premise.
> According to Einstein's conclusion, simultaneity is relative, so the "lightning strikes both ends of the rail simultaneously" in his premise must be conditional. He must specify which observer the "simultaneity" in the premise is relative to.
>
> His Conclusion is against his premise !!!
>
> Again, Einstein's theory of simultaneity is nothing less than a paradox. I would name it Einstein's Paradox. This should be the biggest paradox in physics in the 20th century. (chapter 9 in "absolute time")
>
> for more detail : https://www.amazon.com/Absolute-Time-Relativity-Jack-Liu/dp/B0BQ9JB4RQ
>

I don't see how expect to achieve anything by just making stuff up.
Einstein did not mention lightning in the paper that introduced special
relativity to the world.

Sylvia.

Re: Einstein Paradox: Relativity of Simultaneity

<u2ieve$2rl1n$1@dont-email.me>

  copy mid

https://www.novabbs.com/tech/article-flat.php?id=113915&group=sci.physics.relativity#113915

  copy link   Newsgroups: sci.physics.relativity
Path: i2pn2.org!i2pn.org!eternal-september.org!news.eternal-september.org!.POSTED!not-for-mail
From: vol...@invalid.invalid (Volney)
Newsgroups: sci.physics.relativity
Subject: Re: Einstein Paradox: Relativity of Simultaneity
Date: Sat, 29 Apr 2023 02:54:43 -0400
Organization: A noiseless patient Spider
Lines: 47
Message-ID: <u2ieve$2rl1n$1@dont-email.me>
References: <887d51c1-c60c-4d24-993e-7941d85c4311n@googlegroups.com>
<d2db6956-eba1-4a39-9a09-2a5fbaee5fcdn@googlegroups.com>
<u2gpgn$2gg8j$1@dont-email.me>
<0ec41357-3ba8-4f93-9979-d8d0036fa660n@googlegroups.com>
MIME-Version: 1.0
Content-Type: text/plain; charset=UTF-8; format=flowed
Content-Transfer-Encoding: 8bit
Injection-Date: Sat, 29 Apr 2023 06:54:38 -0000 (UTC)
Injection-Info: dont-email.me; posting-host="5abf68a36468999275cfec4cf545e99d";
logging-data="3003447"; mail-complaints-to="abuse@eternal-september.org"; posting-account="U2FsdGVkX18SDspQoQ121Lo2Vi50jsVr"
User-Agent: Mozilla/5.0 (Windows NT 6.1; Win64; x64; rv:102.0) Gecko/20100101
Thunderbird/102.10.0
Cancel-Lock: sha1:NxYIxTm63NYFdaXEna4TdTb7NwU=
Content-Language: en-US
In-Reply-To: <0ec41357-3ba8-4f93-9979-d8d0036fa660n@googlegroups.com>
 by: Volney - Sat, 29 Apr 2023 06:54 UTC

On 4/28/2023 10:56 PM, gehan.am...@gmail.com wrote:
> On Friday, April 28, 2023 at 8:42:18 PM UTC+5, Volney wrote:
>> On 4/28/2023 3:43 AM, JanPB wrote:
>>> On Tuesday, April 25, 2023 at 2:23:08 PM UTC-7, Jack Liu wrote:
>>>> Einstein's theory of simultaneity is nothing less than a paradox. I would name it Einstein's Paradox. This should be the biggest paradox in physics in the 20th century. (chapter 9 in "absolute time")
>>>
>>> Simultaneity is not essential for SR, it's merely a convenience for
>>> both derivation and use. But not for the theory as such.
>> Also, relativity of simultaneity doesn't depend on length
>> contraction/time dilation in SR in all cases.
>>
>> Consider Einstein's train of length L, but is STATIONARY (stopped).
>> Observer A is on the embankment at the midpoint of the train. Observer B
>> is on the embankment at the front of the train. Observer C is on the
>> embankment at the rear.
>>
>> Observer A sees simultaneous lightning strikes at the front and rear of
>> the train. A says the strikes are simultaneous, as the light from each
>> reaches her at the same time. A observes the strikes L/2c after they
>> actually happen.
>>
>> What does B see? B sees the strike on the front and time L/c later sees
>> the strike on the rear.
>>
>> What does C see? C sees the strike on the rear and time L/c later sees
>> the strike on the front.
>>
>> No SR, no GR, no length contraction or time dilation, nothing but a
>> finite speed of light. But the three observers all disagree on the
>> simultaneity of the lightning strikes.
>
> This is because the person closer to the lightning strikes sees it first.

The point is the three observers all disagree which strike happened
first, with no SR or GR involved whatsoever.
>
> The unstated assumption is that the person on the train does not know he is moving.

I never mentioned anyone on the train!
>
> This is why I call it a thoughtless experiment.

It may be "thoughtless" because you never gave any thought to it.
>
> What about another thought experiment?

Maybe if you understood this one first.

Re: Einstein Paradox: Relativity of Simultaneity

<46ffebc6-83dd-4725-bc2f-ac6ebd42a78bn@googlegroups.com>

  copy mid

https://www.novabbs.com/tech/article-flat.php?id=113917&group=sci.physics.relativity#113917

  copy link   Newsgroups: sci.physics.relativity
X-Received: by 2002:a05:622a:1806:b0:3ef:32db:b249 with SMTP id t6-20020a05622a180600b003ef32dbb249mr3445258qtc.1.1682754648888;
Sat, 29 Apr 2023 00:50:48 -0700 (PDT)
X-Received: by 2002:ac8:7d41:0:b0:3e8:f79d:bdfa with SMTP id
h1-20020ac87d41000000b003e8f79dbdfamr2779114qtb.0.1682754648683; Sat, 29 Apr
2023 00:50:48 -0700 (PDT)
Path: i2pn2.org!i2pn.org!weretis.net!feeder6.news.weretis.net!1.us.feeder.erje.net!feeder.erje.net!diablo1.usenet.blueworldhosting.com!usenet.blueworldhosting.com!diablo2.usenet.blueworldhosting.com!peer02.iad!feed-me.highwinds-media.com!news.highwinds-media.com!news-out.google.com!nntp.google.com!postnews.google.com!google-groups.googlegroups.com!not-for-mail
Newsgroups: sci.physics.relativity
Date: Sat, 29 Apr 2023 00:50:48 -0700 (PDT)
In-Reply-To: <kb3plpFa9hkU1@mid.individual.net>
Injection-Info: google-groups.googlegroups.com; posting-host=165.225.32.109; posting-account=mI08PwoAAAA3Jr-Q4vb20x7RXVfSK_rd
NNTP-Posting-Host: 165.225.32.109
References: <887d51c1-c60c-4d24-993e-7941d85c4311n@googlegroups.com> <kb3plpFa9hkU1@mid.individual.net>
User-Agent: G2/1.0
MIME-Version: 1.0
Message-ID: <46ffebc6-83dd-4725-bc2f-ac6ebd42a78bn@googlegroups.com>
Subject: Re: Einstein Paradox: Relativity of Simultaneity
From: prokaryo...@gmail.com (Prokaryotic Capase Homolog)
Injection-Date: Sat, 29 Apr 2023 07:50:48 +0000
Content-Type: text/plain; charset="UTF-8"
Content-Transfer-Encoding: quoted-printable
X-Received-Bytes: 4724
 by: Prokaryotic Capase H - Sat, 29 Apr 2023 07:50 UTC

On Saturday, April 29, 2023 at 12:53:01 AM UTC-5, Sylvia Else wrote:
> On 26-Apr-23 7:23 am, Jack Liu wrote:
> > Einstein's theory of simultaneity is nothing less than a paradox. I would name it Einstein's Paradox. This should be the biggest paradox in physics in the 20th century. (chapter 9 in "absolute time")
> >
> > How did Einstein analyze simultaneity relativity? He first assumed that two lightning bolts hit the rails at both ends of the railway "simultaneously", and then he analyzed whether the lightning hit the rails "simultaneously" in the perspective of two observers located at that very moment in the center of the rail.
> > The focus is not on whether two observers experience different "simultaneity". The point is that Einstein's reasoning was inconsistent. His conclusion directly denies his premise.
> > According to Einstein's conclusion, simultaneity is relative, so the "lightning strikes both ends of the rail simultaneously" in his premise must be conditional. He must specify which observer the "simultaneity" in the premise is relative to.
> >
> > His Conclusion is against his premise !!!
> >
> > Again, Einstein's theory of simultaneity is nothing less than a paradox.. I would name it Einstein's Paradox. This should be the biggest paradox in physics in the 20th century. (chapter 9 in "absolute time")
> >
> > for more detail : https://www.amazon.com/Absolute-Time-Relativity-Jack-Liu/dp/B0BQ9JB4RQ
> >
> I don't see how expect to achieve anything by just making stuff up.
> Einstein did not mention lightning in the paper that introduced special
> relativity to the world.

The famous thought experiment was found in his popular work,
"Relativity: The Special and General Theory". In this short book,
Einstein translated the formal presentation of his paper into
terms more easily grasped by a wide audience.

To tell the truth, I was bothered by the thought experiment the
first time that I encountered it more than 50 or so years ago.
I struggled with the question,

| What happens if we try to follow what the primed observer sees,
| with the primed frame stationary and the unprimed frame moving?
| Wouldn't we witness the light pulses reaching the primed
| observer simultaneously, and reaching the unprimed observer at
| different times?
| | How can a simple shift in viewpoint reverse the results? Is
| this possibly a paradox that invalidates the gedanken?

It took me years before I worked out the answer.

1) The proper length of the train is LONGER than the proper
distance between the lightning strikes.
2) In the frame of the embankment, the moving train is Lorentz-
contracted so that its length is the same as the distance
between the lightning bolts, which of course when measured in
the frame of the embankment is the proper distance.
3) In the frame of the train, the distance between the lightning
strikes is Lorentz-contracted so that it is less than the
length of the train, which of course when measured in the
frame of the train is its proper length.

Here is an animation. If you blink at the wrong time, you can
miss important events, so be prepared to have to watch it several
times before you see everything.
https://en.wikipedia.org/wiki/File:Train_and_Embankment_Thought_Experiment_And_Its_Inverse.gif

Re: Einstein Paradox: Relativity of Simultaneity

<7a06a5e5-5879-4b4b-b326-e40dd4a4b0bbn@googlegroups.com>

  copy mid

https://www.novabbs.com/tech/article-flat.php?id=113918&group=sci.physics.relativity#113918

  copy link   Newsgroups: sci.physics.relativity
X-Received: by 2002:a05:622a:1806:b0:3e6:6502:16b2 with SMTP id t6-20020a05622a180600b003e6650216b2mr2832823qtc.12.1682759552062;
Sat, 29 Apr 2023 02:12:32 -0700 (PDT)
X-Received: by 2002:a05:6214:14a4:b0:5ef:435e:d25d with SMTP id
bo4-20020a05621414a400b005ef435ed25dmr1464286qvb.2.1682759551813; Sat, 29 Apr
2023 02:12:31 -0700 (PDT)
Path: i2pn2.org!i2pn.org!news.neodome.net!feeder1.feed.usenet.farm!feed.usenet.farm!peer02.ams4!peer.am4.highwinds-media.com!peer02.iad!feed-me.highwinds-media.com!news.highwinds-media.com!news-out.google.com!nntp.google.com!postnews.google.com!google-groups.googlegroups.com!not-for-mail
Newsgroups: sci.physics.relativity
Date: Sat, 29 Apr 2023 02:12:31 -0700 (PDT)
In-Reply-To: <46ffebc6-83dd-4725-bc2f-ac6ebd42a78bn@googlegroups.com>
Injection-Info: google-groups.googlegroups.com; posting-host=165.225.32.109; posting-account=mI08PwoAAAA3Jr-Q4vb20x7RXVfSK_rd
NNTP-Posting-Host: 165.225.32.109
References: <887d51c1-c60c-4d24-993e-7941d85c4311n@googlegroups.com>
<kb3plpFa9hkU1@mid.individual.net> <46ffebc6-83dd-4725-bc2f-ac6ebd42a78bn@googlegroups.com>
User-Agent: G2/1.0
MIME-Version: 1.0
Message-ID: <7a06a5e5-5879-4b4b-b326-e40dd4a4b0bbn@googlegroups.com>
Subject: Re: Einstein Paradox: Relativity of Simultaneity
From: prokaryo...@gmail.com (Prokaryotic Capase Homolog)
Injection-Date: Sat, 29 Apr 2023 09:12:32 +0000
Content-Type: text/plain; charset="UTF-8"
Content-Transfer-Encoding: quoted-printable
X-Received-Bytes: 2028
 by: Prokaryotic Capase H - Sat, 29 Apr 2023 09:12 UTC

On Saturday, April 29, 2023 at 2:50:50 AM UTC-5, Prokaryotic Capase Homolog wrote:

> Here is an animation. If you blink at the wrong time, you can
> miss important events, so be prepared to have to watch it several
> times before you see everything.
> https://en.wikipedia.org/wiki/File:Train_and_Embankment_Thought_Experiment_And_Its_Inverse.gif

After watching Kung Fu Panda, I was inspired to add stop
motion effects to the #2 views so that the events are easier
to see.

If you've looked at this animation before and don't see stop
motion, try clearing your browser cache.

Re: Einstein Paradox: Relativity of Simultaneity

<eb8fbbf4-355c-46fb-96f7-63ce7140f859n@googlegroups.com>

  copy mid

https://www.novabbs.com/tech/article-flat.php?id=113921&group=sci.physics.relativity#113921

  copy link   Newsgroups: sci.physics.relativity
X-Received: by 2002:a05:622a:1006:b0:3f0:a9a0:45e3 with SMTP id d6-20020a05622a100600b003f0a9a045e3mr2936159qte.12.1682761641752;
Sat, 29 Apr 2023 02:47:21 -0700 (PDT)
X-Received: by 2002:a05:622a:1a83:b0:3ef:3395:de6a with SMTP id
s3-20020a05622a1a8300b003ef3395de6amr2952565qtc.6.1682761641471; Sat, 29 Apr
2023 02:47:21 -0700 (PDT)
Path: i2pn2.org!i2pn.org!weretis.net!feeder8.news.weretis.net!proxad.net!feeder1-2.proxad.net!209.85.160.216.MISMATCH!news-out.google.com!nntp.google.com!postnews.google.com!google-groups.googlegroups.com!not-for-mail
Newsgroups: sci.physics.relativity
Date: Sat, 29 Apr 2023 02:47:21 -0700 (PDT)
In-Reply-To: <kb3plpFa9hkU1@mid.individual.net>
Injection-Info: google-groups.googlegroups.com; posting-host=98.197.62.158; posting-account=8fhuCAkAAADWKcWwPpTEbux5C4yu40ip
NNTP-Posting-Host: 98.197.62.158
References: <887d51c1-c60c-4d24-993e-7941d85c4311n@googlegroups.com> <kb3plpFa9hkU1@mid.individual.net>
User-Agent: G2/1.0
MIME-Version: 1.0
Message-ID: <eb8fbbf4-355c-46fb-96f7-63ce7140f859n@googlegroups.com>
Subject: Re: Einstein Paradox: Relativity of Simultaneity
From: liu...@gmail.com (Jack Liu)
Injection-Date: Sat, 29 Apr 2023 09:47:21 +0000
Content-Type: text/plain; charset="UTF-8"
Content-Transfer-Encoding: quoted-printable
 by: Jack Liu - Sat, 29 Apr 2023 09:47 UTC

On Saturday, April 29, 2023 at 12:53:01 AM UTC-5, Sylvia Else wrote:
> On 26-Apr-23 7:23 am, Jack Liu wrote:
> > Einstein's theory of simultaneity is nothing less than a paradox. I would name it Einstein's Paradox. This should be the biggest paradox in physics in the 20th century. (chapter 9 in "absolute time")
> >
> > How did Einstein analyze simultaneity relativity? He first assumed that two lightning bolts hit the rails at both ends of the railway "simultaneously", and then he analyzed whether the lightning hit the rails "simultaneously" in the perspective of two observers located at that very moment in the center of the rail.
> > The focus is not on whether two observers experience different "simultaneity". The point is that Einstein's reasoning was inconsistent. His conclusion directly denies his premise.
> > According to Einstein's conclusion, simultaneity is relative, so the "lightning strikes both ends of the rail simultaneously" in his premise must be conditional. He must specify which observer the "simultaneity" in the premise is relative to.
> >
> > His Conclusion is against his premise !!!
> >
> > Again, Einstein's theory of simultaneity is nothing less than a paradox.. I would name it Einstein's Paradox. This should be the biggest paradox in physics in the 20th century. (chapter 9 in "absolute time")
> >
> > for more detail : https://www.amazon.com/Absolute-Time-Relativity-Jack-Liu/dp/B0BQ9JB4RQ
> >
> I don't see how expect to achieve anything by just making stuff up.
> Einstein did not mention lightning in the paper that introduced special
> relativity to the world.
>
> Sylvia.

https://www.amazon.com/Relativity-Special-General-Original-Version/dp/1542472377
Part 1, Chapter 3-5

Re: Einstein Paradox: Relativity of Simultaneity

<80ac7920-ea80-42f4-adbd-1a14574255aen@googlegroups.com>

  copy mid

https://www.novabbs.com/tech/article-flat.php?id=113922&group=sci.physics.relativity#113922

  copy link   Newsgroups: sci.physics.relativity
X-Received: by 2002:a05:620a:1650:b0:74d:ff4c:45b0 with SMTP id c16-20020a05620a165000b0074dff4c45b0mr1142177qko.6.1682762165893;
Sat, 29 Apr 2023 02:56:05 -0700 (PDT)
X-Received: by 2002:ac8:58c4:0:b0:3e3:8172:ff21 with SMTP id
u4-20020ac858c4000000b003e38172ff21mr2912953qta.8.1682762165646; Sat, 29 Apr
2023 02:56:05 -0700 (PDT)
Path: i2pn2.org!i2pn.org!weretis.net!feeder6.news.weretis.net!usenet.blueworldhosting.com!diablo2.usenet.blueworldhosting.com!peer02.iad!feed-me.highwinds-media.com!news.highwinds-media.com!news-out.google.com!nntp.google.com!postnews.google.com!google-groups.googlegroups.com!not-for-mail
Newsgroups: sci.physics.relativity
Date: Sat, 29 Apr 2023 02:56:05 -0700 (PDT)
In-Reply-To: <46ffebc6-83dd-4725-bc2f-ac6ebd42a78bn@googlegroups.com>
Injection-Info: google-groups.googlegroups.com; posting-host=98.197.62.158; posting-account=8fhuCAkAAADWKcWwPpTEbux5C4yu40ip
NNTP-Posting-Host: 98.197.62.158
References: <887d51c1-c60c-4d24-993e-7941d85c4311n@googlegroups.com>
<kb3plpFa9hkU1@mid.individual.net> <46ffebc6-83dd-4725-bc2f-ac6ebd42a78bn@googlegroups.com>
User-Agent: G2/1.0
MIME-Version: 1.0
Message-ID: <80ac7920-ea80-42f4-adbd-1a14574255aen@googlegroups.com>
Subject: Re: Einstein Paradox: Relativity of Simultaneity
From: liu...@gmail.com (Jack Liu)
Injection-Date: Sat, 29 Apr 2023 09:56:05 +0000
Content-Type: text/plain; charset="UTF-8"
Content-Transfer-Encoding: quoted-printable
X-Received-Bytes: 5306
 by: Jack Liu - Sat, 29 Apr 2023 09:56 UTC

On Saturday, April 29, 2023 at 2:50:50 AM UTC-5, Prokaryotic Capase Homolog wrote:
> On Saturday, April 29, 2023 at 12:53:01 AM UTC-5, Sylvia Else wrote:
> > On 26-Apr-23 7:23 am, Jack Liu wrote:
> > > Einstein's theory of simultaneity is nothing less than a paradox. I would name it Einstein's Paradox. This should be the biggest paradox in physics in the 20th century. (chapter 9 in "absolute time")
> > >
> > > How did Einstein analyze simultaneity relativity? He first assumed that two lightning bolts hit the rails at both ends of the railway "simultaneously", and then he analyzed whether the lightning hit the rails "simultaneously" in the perspective of two observers located at that very moment in the center of the rail.
> > > The focus is not on whether two observers experience different "simultaneity". The point is that Einstein's reasoning was inconsistent. His conclusion directly denies his premise.
> > > According to Einstein's conclusion, simultaneity is relative, so the "lightning strikes both ends of the rail simultaneously" in his premise must be conditional. He must specify which observer the "simultaneity" in the premise is relative to.
> > >
> > > His Conclusion is against his premise !!!
> > >
> > > Again, Einstein's theory of simultaneity is nothing less than a paradox. I would name it Einstein's Paradox. This should be the biggest paradox in physics in the 20th century. (chapter 9 in "absolute time")
> > >
> > > for more detail : https://www.amazon.com/Absolute-Time-Relativity-Jack-Liu/dp/B0BQ9JB4RQ
> > >
> > I don't see how expect to achieve anything by just making stuff up.
> > Einstein did not mention lightning in the paper that introduced special
> > relativity to the world.
> The famous thought experiment was found in his popular work,
> "Relativity: The Special and General Theory". In this short book,
> Einstein translated the formal presentation of his paper into
> terms more easily grasped by a wide audience.
>
> To tell the truth, I was bothered by the thought experiment the
> first time that I encountered it more than 50 or so years ago.
> I struggled with the question,
>
> | What happens if we try to follow what the primed observer sees,
> | with the primed frame stationary and the unprimed frame moving?
> | Wouldn't we witness the light pulses reaching the primed
> | observer simultaneously, and reaching the unprimed observer at
> | different times?
> |
> | How can a simple shift in viewpoint reverse the results? Is
> | this possibly a paradox that invalidates the gedanken?
>
> It took me years before I worked out the answer.
>
> 1) The proper length of the train is LONGER than the proper
> distance between the lightning strikes.
> 2) In the frame of the embankment, the moving train is Lorentz-
> contracted so that its length is the same as the distance
> between the lightning bolts, which of course when measured in
> the frame of the embankment is the proper distance.
> 3) In the frame of the train, the distance between the lightning
> strikes is Lorentz-contracted so that it is less than the
> length of the train, which of course when measured in the
> frame of the train is its proper length.
>
> Here is an animation. If you blink at the wrong time, you can
> miss important events, so be prepared to have to watch it several
> times before you see everything.
> https://en.wikipedia.org/wiki/File:Train_and_Embankment_Thought_Experiment_And_Its_Inverse.gif

How would you work out a answer? that is PARADOX: any conclusion will be against its premise.

In the premise he said two lightning hit road with "simultaneity", the conclusion is two lightning hit road without simultaneity". That is biggest Paradox in Relativity, funnier than Twin Paradox.

Re: Einstein Paradox: Relativity of Simultaneity

<a7886d6c-f90c-415c-a7c2-0a52a5a67850n@googlegroups.com>

  copy mid

https://www.novabbs.com/tech/article-flat.php?id=113923&group=sci.physics.relativity#113923

  copy link   Newsgroups: sci.physics.relativity
X-Received: by 2002:a05:620a:131b:b0:74a:92e:bcfb with SMTP id o27-20020a05620a131b00b0074a092ebcfbmr1426346qkj.3.1682762606951;
Sat, 29 Apr 2023 03:03:26 -0700 (PDT)
X-Received: by 2002:a05:620a:1239:b0:74d:33a7:1049 with SMTP id
v25-20020a05620a123900b0074d33a71049mr1412729qkj.14.1682762606704; Sat, 29
Apr 2023 03:03:26 -0700 (PDT)
Path: i2pn2.org!i2pn.org!weretis.net!feeder6.news.weretis.net!news.misty.com!border-2.nntp.ord.giganews.com!border-1.nntp.ord.giganews.com!nntp.giganews.com!news-out.google.com!nntp.google.com!postnews.google.com!google-groups.googlegroups.com!not-for-mail
Newsgroups: sci.physics.relativity
Date: Sat, 29 Apr 2023 03:03:26 -0700 (PDT)
In-Reply-To: <kb3plpFa9hkU1@mid.individual.net>
Injection-Info: google-groups.googlegroups.com; posting-host=98.197.62.158; posting-account=8fhuCAkAAADWKcWwPpTEbux5C4yu40ip
NNTP-Posting-Host: 98.197.62.158
References: <887d51c1-c60c-4d24-993e-7941d85c4311n@googlegroups.com> <kb3plpFa9hkU1@mid.individual.net>
User-Agent: G2/1.0
MIME-Version: 1.0
Message-ID: <a7886d6c-f90c-415c-a7c2-0a52a5a67850n@googlegroups.com>
Subject: Re: Einstein Paradox: Relativity of Simultaneity
From: liu...@gmail.com (Jack Liu)
Injection-Date: Sat, 29 Apr 2023 10:03:26 +0000
Content-Type: text/plain; charset="UTF-8"
Content-Transfer-Encoding: quoted-printable
Lines: 51
 by: Jack Liu - Sat, 29 Apr 2023 10:03 UTC

On Saturday, April 29, 2023 at 12:53:01 AM UTC-5, Sylvia Else wrote:
> On 26-Apr-23 7:23 am, Jack Liu wrote:
> > Einstein's theory of simultaneity is nothing less than a paradox. I would name it Einstein's Paradox. This should be the biggest paradox in physics in the 20th century. (chapter 9 in "absolute time")
> >
> > How did Einstein analyze simultaneity relativity? He first assumed that two lightning bolts hit the rails at both ends of the railway "simultaneously", and then he analyzed whether the lightning hit the rails "simultaneously" in the perspective of two observers located at that very moment in the center of the rail.
> > The focus is not on whether two observers experience different "simultaneity". The point is that Einstein's reasoning was inconsistent. His conclusion directly denies his premise.
> > According to Einstein's conclusion, simultaneity is relative, so the "lightning strikes both ends of the rail simultaneously" in his premise must be conditional. He must specify which observer the "simultaneity" in the premise is relative to.
> >
> > His Conclusion is against his premise !!!
> >
> > Again, Einstein's theory of simultaneity is nothing less than a paradox.. I would name it Einstein's Paradox. This should be the biggest paradox in physics in the 20th century. (chapter 9 in "absolute time")
> >
> > for more detail : https://www.amazon.com/Absolute-Time-Relativity-Jack-Liu/dp/B0BQ9JB4RQ
> >
> I don't see how expect to achieve anything by just making stuff up.
> Einstein did not mention lightning in the paper that introduced special
> relativity to the world.
>
> Sylvia.

To Sylvia.

I don't expect you agree with me for anything. But I agree with you for the following 2 points :

1. that I am the one making stuff up, I am the one to name it Einstein Paradox.
2. that you said Einstein did not mention lightning in the paper that introduced special relativity to the world, which means you don't care it is Paradox since Einstein did not mention lightning in the paper that introduced special relativity.

That is good enough. Seems we have finally some agreement.

Jack

Re: Einstein Paradox: Relativity of Simultaneity

<u2j0pn$2ubnr$2@dont-email.me>

  copy mid

https://www.novabbs.com/tech/article-flat.php?id=113930&group=sci.physics.relativity#113930

  copy link   Newsgroups: sci.physics.relativity sci.physics sci.math
Path: i2pn2.org!i2pn.org!eternal-september.org!news.eternal-september.org!.POSTED!not-for-mail
From: veo...@rnigeern.ne (Evenezer Nigro)
Newsgroups: sci.physics.relativity,sci.physics,sci.math
Subject: Re: Einstein Paradox: Relativity of Simultaneity
Date: Sat, 29 Apr 2023 11:58:47 -0000 (UTC)
Organization: A noiseless patient Spider
Lines: 23
Message-ID: <u2j0pn$2ubnr$2@dont-email.me>
References: <887d51c1-c60c-4d24-993e-7941d85c4311n@googlegroups.com>
<kb3plpFa9hkU1@mid.individual.net>
MIME-Version: 1.0
Content-Type: text/plain; charset=UTF-8
Content-Transfer-Encoding: 8bit
Injection-Date: Sat, 29 Apr 2023 11:58:47 -0000 (UTC)
Injection-Info: dont-email.me; posting-host="298011f6388ec226eaa73207991bcfd7";
logging-data="3092219"; mail-complaints-to="abuse@eternal-september.org"; posting-account="U2FsdGVkX1/RpeuWsJhmI/wvaCExfVWE"
User-Agent: Mozilla 3.04Gold (WinNT; U)
Cancel-Lock: sha1:rbeLVxAtGq6xRVX1CLcCKRmDDek=
X-Face: %i[?pV)on.A\|N+`/EXZHMG;UN9};!ScPL*.AoJI\W\<KpJ/,V6-&4x]&&<XWrwM
DtRx;nOt2i2Ca7RhFj6So#lCv%00>O{OCUs'89g+_}KWSEZz'/8)=2FrLD%iI-KX48tIusD
JBOiz["&P]j0-:7i_UDdCm@`W/3APh4iK~LA`J.zMg{^`O#@q5*bF/!w6WfidF=RE=A0-Q"
IG2Mx$k!9ukS?[v7<5$IM4rWL$8Yjh<q]RGQS{#j4w$MH?TAVjpOz)
Face: iVBORw0KGgoAAAANSUhEUgAAADAAAAAwBAMAAAClLOS0AAAAElBMVEW5c17b1M3M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 by: Evenezer Nigro - Sat, 29 Apr 2023 11:58 UTC

Sylvia Else wrote:

> I don't see how expect to achieve anything by just making stuff up.
> Einstein did not mention lightning in the paper that introduced special
> relativity to the world.

you mean, his wife Mileva. His wife, not his cousin, he was fooling around
with.

Citizenship
Kingdom of Württemberg, part of the German Empire (until 1896)
Stateless (1896–1901)
Switzerland (1901–1955)
Austria, part of the Austro-Hungarian Empire (1911–1912)
Kingdom of Prussia, part of the German Empire (1914–1918)[note 1]
Free State of Prussia (Weimar Republic, 1918–1933)
United States (from 1940)
Education
Federal polytechnic school in Zurich (Federal teaching diploma, 1900)
University of Zurich (PhD, 1905)

which means, that his wife *_made_him_with_that_PhD_diploma_1905_*. My
butt, it looks like this man was *_a_true_fraudster_*.

Re: Einstein Paradox: Relativity of Simultaneity

<dJUEdrNPwR_D5ff0989WYMeINQE@jntp>

  copy mid

https://www.novabbs.com/tech/article-flat.php?id=113931&group=sci.physics.relativity#113931

  copy link   Newsgroups: sci.physics.relativity
Path: i2pn2.org!i2pn.org!paganini.bofh.team!pasdenom.info!from-devjntp
Message-ID: <dJUEdrNPwR_D5ff0989WYMeINQE@jntp>
JNTP-Route: news2.nemoweb.net
JNTP-DataType: Article
Subject: Re: Einstein Paradox: Relativity of Simultaneity
References: <887d51c1-c60c-4d24-993e-7941d85c4311n@googlegroups.com> <d2db6956-eba1-4a39-9a09-2a5fbaee5fcdn@googlegroups.com>
<u2gpgn$2gg8j$1@dont-email.me> <5cdada26-5b79-4620-8ac1-7602bf5442c5n@googlegroups.com>
<c87d1b2f-9e7c-4411-81cd-20a2f271c8c4n@googlegroups.com> <0747548f-bc47-46d7-b0c2-68d09c3a7e9an@googlegroups.com>
<e51ef3fa-5e52-48e6-bf4a-3f8499f876adn@googlegroups.com> <_5CJAa4qjAloBDuFOEVK0NIA2m0@jntp>
<35fc3de8-b325-4be7-804d-1e3364fe1dd7n@googlegroups.com>
Newsgroups: sci.physics.relativity
JNTP-HashClient: M012cqwHSc0dadi4hUIeX3jepmg
JNTP-ThreadID: 887d51c1-c60c-4d24-993e-7941d85c4311n@googlegroups.com
JNTP-Uri: http://news2.nemoweb.net/?DataID=dJUEdrNPwR_D5ff0989WYMeINQE@jntp
User-Agent: Nemo/0.999a
JNTP-OriginServer: news2.nemoweb.net
Date: Sat, 29 Apr 23 12:01:31 +0000
Organization: Nemoweb
JNTP-Browser: Mozilla/5.0 (Windows NT 10.0; Win64; x64) AppleWebKit/537.36 (KHTML, like Gecko) Chrome/112.0.0.0 Safari/537.36
Injection-Info: news2.nemoweb.net; posting-host="34f88a401fb9c9d6baaca65ef73aa9a2fa197877"; logging-data="2023-04-29T12:01:31Z/7884204"; posting-account="4@news2.nemoweb.net"; mail-complaints-to="newsmaster@news2.nemoweb.net"
JNTP-ProtocolVersion: 0.21.1
JNTP-Server: PhpNemoServer/0.94.5
MIME-Version: 1.0
Content-Type: text/plain; charset=UTF-8; format=flowed
Content-Transfer-Encoding: 8bit
X-JNTP-JsonNewsGateway: 0.96
From: r.hac...@jesaispu.fr (Richard Hachel)
 by: Richard Hachel - Sat, 29 Apr 2023 12:01 UTC

Le 29/04/2023 à 02:38, Jack Liu a écrit :
>
> Dear R. H.
>
> Thank you for reading my book.
>
> That is Lorentz Factor I derive for inbound motion Lorentz Transformation.
>
> Einstein consider only outbound moving body and derive a Lorentz factor > 0,
> which indicate time dilation. That is only half of the whole picture.
> I apply same logic of Einstein to apply to inbound moving body, and derive to
> above new Lorentz factor which is <1, which indicate time contraction.
>
> I try to demonstrate that, once SP continue to develop another half, it will
> contradict to itself.
>

I don't understand your use of the "+" sign in your equation.

The "1/sqrt(1-v²/c²)" factor proposed by Henri Poincaré (the greatest
mathematician in the history of mankind and the only one who was able to
control the entirety of all the science of his time ( physics,
mathematics, philosophy) which would no longer be possible today as the
fields have become vast) is correct.

You must ask:
Vo=Vr/sqrt(1+Vr²/c²)

and Vr=Vo/sqrt(1-Vo²/c²)

These are the two reciprocal equations.

So we have g=1/sqrt(1-Vo²/c²)

or g=sqrt(1+Vr²/c²)

But g=1/sqrt(1+v²/c²) doesn't seem to be of interest.

R.H.

Re: Einstein Paradox: Relativity of Simultaneity

<u2j1ju$2ubnr$3@dont-email.me>

  copy mid

https://www.novabbs.com/tech/article-flat.php?id=113933&group=sci.physics.relativity#113933

  copy link   Newsgroups: sci.physics.relativity sci.physics sci.math
Path: i2pn2.org!i2pn.org!eternal-september.org!news.eternal-september.org!.POSTED!not-for-mail
From: veo...@rnigeern.ne (Evenezer Nigro)
Newsgroups: sci.physics.relativity,sci.physics,sci.math
Subject: Re: Einstein Paradox: Relativity of Simultaneity
Date: Sat, 29 Apr 2023 12:12:46 -0000 (UTC)
Organization: A noiseless patient Spider
Lines: 19
Message-ID: <u2j1ju$2ubnr$3@dont-email.me>
References: <887d51c1-c60c-4d24-993e-7941d85c4311n@googlegroups.com>
<kb3plpFa9hkU1@mid.individual.net>
MIME-Version: 1.0
Content-Type: text/plain; charset=UTF-8
Content-Transfer-Encoding: 8bit
Injection-Date: Sat, 29 Apr 2023 12:12:46 -0000 (UTC)
Injection-Info: dont-email.me; posting-host="298011f6388ec226eaa73207991bcfd7";
logging-data="3092219"; mail-complaints-to="abuse@eternal-september.org"; posting-account="U2FsdGVkX1+v9fKzPytVYwNFT/OdcYWH"
User-Agent: MesNews/1.08.06.00-gb
Cancel-Lock: sha1:gH3bM17RBBjIapxvk29qr5uhcDw=
Face: iVBORw0KGgoAAAANSUhEUgAAADAAAAAwBAMAAAClLOS0AAAAGFBMVEXIeHUCBQkm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X-Face: 6{(9Cc%u'_1aH6J/SlRb('-b=Hi+u<0d1oj5xl*m*>{XlW/s1a?w*[%~?_VP]ceD
3"T}SLs;Qp@iP9=9;Qlsm.i-U/{U'l~{|qBPA3>0jihMr(Cn*`BL)e>+Ho3XK5Ke<JGiVx#
Z_*oObKnfkt:l!-.WtU~)/h5=y"ev}cybv>~50c=L(;#WqK"Q%:RshXf0FZ+d$n`(#'G*`t
f;43Ph'gY"IS[dkn6]e/ZY8C6bqF_#aCMe]I)t"6]DmGCCU30XAU/&d=jg}]T]69ZAFfN&k zg)g
 by: Evenezer Nigro - Sat, 29 Apr 2023 12:12 UTC

Sylvia Else wrote:

>> for more detail :
>> https://www.amazon.com/Absolute-Time-Relativity-Jack-Liu/dp/B0BQ9JB4RQ
>>
>>
> I don't see how expect to achieve anything by just making stuff up.
> Einstein did not mention lightning in the paper that introduced special
> relativity to the world.

https://en.wikipedia.org/wiki/Albert_Einstein#1921–1922:_Travels_abroad

Einstein with his second wife, Elsa, his cousin, in 1921
https://en.wikipedia.org/wiki/Albert_Einstein#/media/File:Einstein_Albert_Elsa_LOC_32096u.jpg

can't even believe what I read.
https://en.wikipedia.org/wiki/Albert_Einstein#Marriages_and_children

Einstein's chief romantic attraction was now his cousin Elsa Löwenthal;[48] she was his *_first_cousin_maternally_* and *_second_cousin_paternally_*.[49] Einstein and Marić divorced on 14 February 1919, having lived apart for five years.[50][51] As part of the divorce settlement, Einstein agreed to give Marić any future (in the event, 1921) *_Nobel_Prize_money_*.[52]

Re: Einstein Paradox: Relativity of Simultaneity

<kb4gsbFdn9fU1@mid.individual.net>

  copy mid

https://www.novabbs.com/tech/article-flat.php?id=113938&group=sci.physics.relativity#113938

  copy link   Newsgroups: sci.physics.relativity
Path: i2pn2.org!i2pn.org!weretis.net!feeder8.news.weretis.net!news.imp.ch!fu-berlin.de!uni-berlin.de!individual.net!not-for-mail
From: syl...@email.invalid (Sylvia Else)
Newsgroups: sci.physics.relativity
Subject: Re: Einstein Paradox: Relativity of Simultaneity
Date: Sat, 29 Apr 2023 22:28:59 +1000
Lines: 36
Message-ID: <kb4gsbFdn9fU1@mid.individual.net>
References: <887d51c1-c60c-4d24-993e-7941d85c4311n@googlegroups.com>
<kb3plpFa9hkU1@mid.individual.net>
<a7886d6c-f90c-415c-a7c2-0a52a5a67850n@googlegroups.com>
Mime-Version: 1.0
Content-Type: text/plain; charset=UTF-8; format=flowed
Content-Transfer-Encoding: 8bit
X-Trace: individual.net HXuPeuIZ/4qMX8cDVWbrwQKr2baJIJpKeKgYPyCObYPisdTgHm
Cancel-Lock: sha1:ayrmqzXEF3rBx8t7NkXuRqm6h7U=
User-Agent: Mozilla/5.0 (Windows NT 10.0; Win64; x64; rv:102.0) Gecko/20100101
Thunderbird/102.10.1
Content-Language: en-GB
In-Reply-To: <a7886d6c-f90c-415c-a7c2-0a52a5a67850n@googlegroups.com>
 by: Sylvia Else - Sat, 29 Apr 2023 12:28 UTC

On 29-Apr-23 8:03 pm, Jack Liu wrote:
> On Saturday, April 29, 2023 at 12:53:01 AM UTC-5, Sylvia Else wrote:
>> On 26-Apr-23 7:23 am, Jack Liu wrote:
>>> Einstein's theory of simultaneity is nothing less than a paradox. I would name it Einstein's Paradox. This should be the biggest paradox in physics in the 20th century. (chapter 9 in "absolute time")
>>>
>>> How did Einstein analyze simultaneity relativity? He first assumed that two lightning bolts hit the rails at both ends of the railway "simultaneously", and then he analyzed whether the lightning hit the rails "simultaneously" in the perspective of two observers located at that very moment in the center of the rail.
>>> The focus is not on whether two observers experience different "simultaneity". The point is that Einstein's reasoning was inconsistent. His conclusion directly denies his premise.
>>> According to Einstein's conclusion, simultaneity is relative, so the "lightning strikes both ends of the rail simultaneously" in his premise must be conditional. He must specify which observer the "simultaneity" in the premise is relative to.
>>>
>>> His Conclusion is against his premise !!!
>>>
>>> Again, Einstein's theory of simultaneity is nothing less than a paradox. I would name it Einstein's Paradox. This should be the biggest paradox in physics in the 20th century. (chapter 9 in "absolute time")
>>>
>>> for more detail : https://www.amazon.com/Absolute-Time-Relativity-Jack-Liu/dp/B0BQ9JB4RQ
>>>
>> I don't see how expect to achieve anything by just making stuff up.
>> Einstein did not mention lightning in the paper that introduced special
>> relativity to the world.
>>
>> Sylvia.
>
> To Sylvia.
>
> I don't expect you agree with me for anything. But I agree with you for the following 2 points :
>
> 1. that I am the one making stuff up, I am the one to name it Einstein Paradox.
> 2. that you said Einstein did not mention lightning in the paper that introduced special relativity to the world, which means you don't care it is Paradox since Einstein did not mention lightning in the paper that introduced special relativity.
>
> That is good enough. Seems we have finally some agreement.
>
> Jack
>

No, it means that you should not invent straw men.

Sylvia.

Re: Einstein Paradox: Relativity of Simultaneity

<b3242fb1-4ce2-40ba-874d-df2cf0385dcen@googlegroups.com>

  copy mid

https://www.novabbs.com/tech/article-flat.php?id=113944&group=sci.physics.relativity#113944

  copy link   Newsgroups: sci.physics.relativity
X-Received: by 2002:a05:620a:1477:b0:74e:e9a:bec2 with SMTP id j23-20020a05620a147700b0074e0e9abec2mr1475443qkl.5.1682772069288;
Sat, 29 Apr 2023 05:41:09 -0700 (PDT)
X-Received: by 2002:a05:622a:1806:b0:3ef:32db:b249 with SMTP id
t6-20020a05622a180600b003ef32dbb249mr3584644qtc.1.1682772069028; Sat, 29 Apr
2023 05:41:09 -0700 (PDT)
Path: i2pn2.org!i2pn.org!weretis.net!feeder8.news.weretis.net!proxad.net!feeder1-2.proxad.net!209.85.160.216.MISMATCH!news-out.google.com!nntp.google.com!postnews.google.com!google-groups.googlegroups.com!not-for-mail
Newsgroups: sci.physics.relativity
Date: Sat, 29 Apr 2023 05:41:08 -0700 (PDT)
In-Reply-To: <u2ieve$2rl1n$1@dont-email.me>
Injection-Info: google-groups.googlegroups.com; posting-host=185.215.32.55; posting-account=sVBCDQoAAAADe-Ogi2R38m91EmLrcIgt
NNTP-Posting-Host: 185.215.32.55
References: <887d51c1-c60c-4d24-993e-7941d85c4311n@googlegroups.com>
<d2db6956-eba1-4a39-9a09-2a5fbaee5fcdn@googlegroups.com> <u2gpgn$2gg8j$1@dont-email.me>
<0ec41357-3ba8-4f93-9979-d8d0036fa660n@googlegroups.com> <u2ieve$2rl1n$1@dont-email.me>
User-Agent: G2/1.0
MIME-Version: 1.0
Message-ID: <b3242fb1-4ce2-40ba-874d-df2cf0385dcen@googlegroups.com>
Subject: Re: Einstein Paradox: Relativity of Simultaneity
From: gehan.am...@gmail.com (gehan.am...@gmail.com)
Injection-Date: Sat, 29 Apr 2023 12:41:09 +0000
Content-Type: text/plain; charset="UTF-8"
Content-Transfer-Encoding: quoted-printable
 by: gehan.am...@gmail.co - Sat, 29 Apr 2023 12:41 UTC

On Saturday, April 29, 2023 at 11:54:41 AM UTC+5, Volney wrote:
> On 4/28/2023 10:56 PM, gehan.am...@gmail.com wrote:
> > On Friday, April 28, 2023 at 8:42:18 PM UTC+5, Volney wrote:
> >> On 4/28/2023 3:43 AM, JanPB wrote:
> >>> On Tuesday, April 25, 2023 at 2:23:08 PM UTC-7, Jack Liu wrote:
> >>>> Einstein's theory of simultaneity is nothing less than a paradox. I would name it Einstein's Paradox. This should be the biggest paradox in physics in the 20th century. (chapter 9 in "absolute time")
> >>>
> >>> Simultaneity is not essential for SR, it's merely a convenience for
> >>> both derivation and use. But not for the theory as such.
> >> Also, relativity of simultaneity doesn't depend on length
> >> contraction/time dilation in SR in all cases.
> >>
> >> Consider Einstein's train of length L, but is STATIONARY (stopped).
> >> Observer A is on the embankment at the midpoint of the train. Observer B
> >> is on the embankment at the front of the train. Observer C is on the
> >> embankment at the rear.
> >>
> >> Observer A sees simultaneous lightning strikes at the front and rear of
> >> the train. A says the strikes are simultaneous, as the light from each
> >> reaches her at the same time. A observes the strikes L/2c after they
> >> actually happen.
> >>
> >> What does B see? B sees the strike on the front and time L/c later sees
> >> the strike on the rear.
> >>
> >> What does C see? C sees the strike on the rear and time L/c later sees
> >> the strike on the front.
> >>
> >> No SR, no GR, no length contraction or time dilation, nothing but a
> >> finite speed of light. But the three observers all disagree on the
> >> simultaneity of the lightning strikes.
> >
> > This is because the person closer to the lightning strikes sees it first.
> The point is the three observers all disagree which strike happened
> first, with no SR or GR involved whatsoever.
> >
> > The unstated assumption is that the person on the train does not know he is moving.
> I never mentioned anyone on the train!
> >
You are right of course. So what does this all mean?

Why does Einstein use this example?

Re: Einstein Paradox: Relativity of Simultaneity

<53043133-a572-4e87-83b9-8576a8c7ee37n@googlegroups.com>

  copy mid

https://www.novabbs.com/tech/article-flat.php?id=113946&group=sci.physics.relativity#113946

  copy link   Newsgroups: sci.physics.relativity
X-Received: by 2002:ac8:4e8e:0:b0:3ef:3b04:b8e2 with SMTP id 14-20020ac84e8e000000b003ef3b04b8e2mr2992112qtp.0.1682772318885;
Sat, 29 Apr 2023 05:45:18 -0700 (PDT)
X-Received: by 2002:ac8:5e0a:0:b0:3ef:41a9:7db1 with SMTP id
h10-20020ac85e0a000000b003ef41a97db1mr2373650qtx.2.1682772318594; Sat, 29 Apr
2023 05:45:18 -0700 (PDT)
Path: i2pn2.org!i2pn.org!weretis.net!feeder6.news.weretis.net!usenet.blueworldhosting.com!diablo2.usenet.blueworldhosting.com!peer02.iad!feed-me.highwinds-media.com!news.highwinds-media.com!news-out.google.com!nntp.google.com!postnews.google.com!google-groups.googlegroups.com!not-for-mail
Newsgroups: sci.physics.relativity
Date: Sat, 29 Apr 2023 05:45:18 -0700 (PDT)
In-Reply-To: <46ffebc6-83dd-4725-bc2f-ac6ebd42a78bn@googlegroups.com>
Injection-Info: google-groups.googlegroups.com; posting-host=185.215.32.55; posting-account=sVBCDQoAAAADe-Ogi2R38m91EmLrcIgt
NNTP-Posting-Host: 185.215.32.55
References: <887d51c1-c60c-4d24-993e-7941d85c4311n@googlegroups.com>
<kb3plpFa9hkU1@mid.individual.net> <46ffebc6-83dd-4725-bc2f-ac6ebd42a78bn@googlegroups.com>
User-Agent: G2/1.0
MIME-Version: 1.0
Message-ID: <53043133-a572-4e87-83b9-8576a8c7ee37n@googlegroups.com>
Subject: Re: Einstein Paradox: Relativity of Simultaneity
From: gehan.am...@gmail.com (gehan.am...@gmail.com)
Injection-Date: Sat, 29 Apr 2023 12:45:18 +0000
Content-Type: text/plain; charset="UTF-8"
Content-Transfer-Encoding: quoted-printable
X-Received-Bytes: 5254
 by: gehan.am...@gmail.co - Sat, 29 Apr 2023 12:45 UTC

On Saturday, April 29, 2023 at 12:50:50 PM UTC+5, Prokaryotic Capase Homolog wrote:
> On Saturday, April 29, 2023 at 12:53:01 AM UTC-5, Sylvia Else wrote:
> > On 26-Apr-23 7:23 am, Jack Liu wrote:
> > > Einstein's theory of simultaneity is nothing less than a paradox. I would name it Einstein's Paradox. This should be the biggest paradox in physics in the 20th century. (chapter 9 in "absolute time")
> > >
> > > How did Einstein analyze simultaneity relativity? He first assumed that two lightning bolts hit the rails at both ends of the railway "simultaneously", and then he analyzed whether the lightning hit the rails "simultaneously" in the perspective of two observers located at that very moment in the center of the rail.
> > > The focus is not on whether two observers experience different "simultaneity". The point is that Einstein's reasoning was inconsistent. His conclusion directly denies his premise.
> > > According to Einstein's conclusion, simultaneity is relative, so the "lightning strikes both ends of the rail simultaneously" in his premise must be conditional. He must specify which observer the "simultaneity" in the premise is relative to.
> > >
> > > His Conclusion is against his premise !!!
> > >
> > > Again, Einstein's theory of simultaneity is nothing less than a paradox. I would name it Einstein's Paradox. This should be the biggest paradox in physics in the 20th century. (chapter 9 in "absolute time")
> > >
> > > for more detail : https://www.amazon.com/Absolute-Time-Relativity-Jack-Liu/dp/B0BQ9JB4RQ
> > >
> > I don't see how expect to achieve anything by just making stuff up.
> > Einstein did not mention lightning in the paper that introduced special
> > relativity to the world.
> The famous thought experiment was found in his popular work,
> "Relativity: The Special and General Theory". In this short book,
> Einstein translated the formal presentation of his paper into
> terms more easily grasped by a wide audience.
>
> To tell the truth, I was bothered by the thought experiment the
> first time that I encountered it more than 50 or so years ago.
> I struggled with the question,
>
> | What happens if we try to follow what the primed observer sees,
> | with the primed frame stationary and the unprimed frame moving?
> | Wouldn't we witness the light pulses reaching the primed
> | observer simultaneously, and reaching the unprimed observer at
> | different times?
> |
> | How can a simple shift in viewpoint reverse the results? Is
> | this possibly a paradox that invalidates the gedanken?
>
> It took me years before I worked out the answer.
>
> 1) The proper length of the train is LONGER than the proper
> distance between the lightning strikes.
> 2) In the frame of the embankment, the moving train is Lorentz-
> contracted so that its length is the same as the distance
> between the lightning bolts, which of course when measured in
> the frame of the embankment is the proper distance.
> 3) In the frame of the train, the distance between the lightning
> strikes is Lorentz-contracted so that it is less than the
> length of the train, which of course when measured in the
> frame of the train is its proper length.
>
> Here is an animation. If you blink at the wrong time, you can
> miss important events, so be prepared to have to watch it several
> times before you see everything.
> https://en.wikipedia.org/wiki/File:Train_and_Embankment_Thought_Experiment_And_Its_Inverse.gif

There is no mention of length contraction in the book.

The experiment no longer bothers me because it is an exact description of what would happen if there is an Aether in the frame of the tracks.

At least agree on this.

Re: Einstein Paradox: Relativity of Simultaneity

<u2j47q$2ubnr$5@dont-email.me>

  copy mid

https://www.novabbs.com/tech/article-flat.php?id=113952&group=sci.physics.relativity#113952

  copy link   Newsgroups: sci.physics.relativity sci.physics sci.math
Path: i2pn2.org!i2pn.org!eternal-september.org!news.eternal-september.org!.POSTED!not-for-mail
From: veo...@rnigeern.ne (Evenezer Nigro)
Newsgroups: sci.physics.relativity,sci.physics,sci.math
Subject: Re: Einstein Paradox: Relativity of Simultaneity
Date: Sat, 29 Apr 2023 12:57:30 -0000 (UTC)
Organization: A noiseless patient Spider
Lines: 270
Message-ID: <u2j47q$2ubnr$5@dont-email.me>
References: <887d51c1-c60c-4d24-993e-7941d85c4311n@googlegroups.com>
<d2db6956-eba1-4a39-9a09-2a5fbaee5fcdn@googlegroups.com>
<u2gpgn$2gg8j$1@dont-email.me>
<3415ec27-f4bd-45dd-8b3c-10711c462eccn@googlegroups.com>
<u2hrtq$2lh6h$1@dont-email.me>
MIME-Version: 1.0
Content-Type: text/plain; charset=UTF-8
Content-Transfer-Encoding: 8bit
Injection-Date: Sat, 29 Apr 2023 12:57:30 -0000 (UTC)
Injection-Info: dont-email.me; posting-host="298011f6388ec226eaa73207991bcfd7";
logging-data="3092219"; mail-complaints-to="abuse@eternal-september.org"; posting-account="U2FsdGVkX18T0EpaCCcc/JW6ofdB1pxt"
User-Agent: Evolution/2.32.3 (Windows NT 5.1; rv:52.0)
Cancel-Lock: sha1:fDdhYxXQXH5PeVebDqKHaAoo0hs=
X-Face: /?Jwzj;[bpBW^cQ4w_'LwcWLKs\S'0y4:hJXgez&Ms;IAv;?y?[m6AQQ'm\*pH3p
1@N:Jd[v[dNS\ZA<e,(:2I.,m}J5/jBM),C@Z4N$#Z+|^nn{_js%0GD,=_>'z+K@wH.TN]P
jL$%p4Za4T2]WiNKB0%"Y975c,lz%amZ-(<;bONo|"a&?$\j9=.kXT"Jk[^SX`U|R%Mp'[N
HBgdPW]!Y8Ou3yNq/S7]Y\RO')%0bu+Rre|$tX-W]kuDrK9DKMzmvPAEL8.f'9%)O=k&l=g
64FJkz';8Z&jK5uI6k.A_^c}&^W(/+g
Face: iVBORw0KGgoAAAANSUhEUgAAADAAAAAwBAMAAAClLOS0AAAAGFBMVEWQepSjsa08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 by: Evenezer Nigro - Sat, 29 Apr 2023 12:57 UTC

Volney wrote:

> On 4/28/2023 12:15 PM, Jack Liu wrote:
>> You are right A, B, C has different about the timing of lightning,
>> about the simultaneity of two event. However Einstein just believes in
>> absolute time by claiming two lightning strike the two end of train
>> SAME TIME at his premise.
>
> No, Einstein stated Observer M at the midpoint on the embankment SEES
> the flashes at the same time. That simply means the light from the two
> flashes enters his eyes at the same time. Because of this M concludes
> the flashes are simultaneous

Secret Team: The Nord Stream Pipeline Sabotage Revisited
https://2%31%73%74%63%65nturywire.com/2023/04/25/secret-team-the-nord-stream-pipeline-sabotage-revisited/

Despite their proximity to the event question, and being a primary stakeholder in the Nord Stream pipeline project, the German government has done little if anything in terms of a serious investigation into the unprecedented attack, with the German officials serving us a remake of the 9/11 story, albeit with a James Bond feel to it, involving fake Bulgarian passports and some trace of explosives on a 50-foot pleasure craft called ‘The Andromeda’. In his follow-up piece entitled, “The Cover-up,“ published on March 22nd, Hersh maintains this was a desperate cover story conjured by US intelligence in order to deflect from his bombshell revelations. According to the Germans, six men are believed to have planted C4 explosives on the seabed at a depth of 262 feet who, and when detonated, this triggered a blast registering 2.5 on the Richter scale. I am confident Hollywood will take full advantage of this compelling plot in combination with the even more spectacular claim from a US intelligence report suggesting that a ‘rogue’ pro-Ukrainian group carried out the attack on the Nord Stream pipelines. A blockbuster in the making, for sure.

Moreover, when the issue was raised at the UN Security Council in late March by the representative of the Russian Federation, both the US and UK voted against an investigation into the Nord Stream bombing.

It’s now time to fill-in the remaining gaps of Hersh’s bombshell story, and reveal the ‘secret team’ who carried out this historic attack on a vital piece of European energy infrastructure. In this article, we will show you which state actors had the means, the motive and the opportunity to carry out this crime. We will also show you the actual demolition diving team that participated in BALTOPS-22, a 13-day naval exercise which featured some 47 ships, 89 aircraft, and 7,000 personnel in the Baltic Sea, under the command of NATO Supreme Allied Commander Europe’s (SACEUR). We will also break down some of the equipment – mini-subs and multi-role support vessels – used by these elite deep sea divers, whilst also identifying the various factions involved in the planning and the execution of what was a well-rehearsed and successful CIA-backed covert operation.

Seymour Hersh has stated that under the cover of BALTOPS-22, C4 explosives were attached to the pipelines, which would later be triggered by a sonar buoy dropped by a military plane. Hersh explains what happened next:

“Once in place, the delayed timing devices attached to any of the four pipelines could be accidentally triggered by the complex mix of ocean background noises throughout the heavily trafficked Baltic Sea—from near and distant ships, underwater drilling, seismic events, waves and even sea creatures. To avoid this, the sonar buoy, once in place, would emit a sequence of unique low frequency tonal sounds—much like those emitted by a flute or a piano—that would be recognized by the timing device and, after a pre-set hours of delay, trigger the explosives.”

The explosions occurred on Sept 26th, and subsequent underwater gas leaks occurred which visibly bubbled to the ocean surface (image, below).

The initial story published by Seymour Hersh did open the door to a plausible explanation as to what had taken place in the Baltic Sea last autumn, but it has also left a number of unanswered questions as to the specifics of the operation. As per his confidential source, Hersh stated that US Navy divers, operating under the cover of the annual NATO exercise known as BALTOPS-22, had planted triggered explosives which were then detonated three months later, destroying three of the four pipelines in the Nord Stream 1 and 2 transit network which carries Russian natural gas to Germany. His source also replayed how this operation planning was led by Biden’s National Security advisor, Jake Sullivan, receiving input from men and women from the Joint Chiefs of Staff, CIA, State and Treasury departments. The initial meetings took place in “a secure room on a top floor of the Old Executive Office Building, adjacent to the White House, that was also the home of the President’s Foreign Intelligence Advisory Board (PFIAB).”

Soon afterwards, “[CIA director] Burns quickly authorized an Agency working group whose ad hoc members included—by chance—someone who was familiar with the capabilities of the Navy’s deep-sea divers in Panama City. Over the next few weeks, members of the CIA’s working group began to craft a plan for a covert operation that would use deep-sea divers to trigger an explosion along the pipeline.” Regarding the special diving operation, Hersh states:

“The divers were Navy only, and not members of America’s Special Operations Command, whose covert operations must be reported to Congress and briefed in advance to the Senate and House leadership—the so-called Gang of Eight. The Biden Administration was doing everything possible to avoid leaks as the planning took place late in 2021 and into the first months of 2022.”

Hersh notes that the idea of a CIA-led special operations using US Navy divers to intervene with Russia infrastructure has solid pedigree. In 1971, Operation Ivy Bells was a joint US Navy and CIA covert mission, which successfully located and wire-tapped an undersea communications cable in 400 feet of water off the coast of the Kamchatka Peninsula located some 2,300 km northeast of Vladivostok in the Russian far east region.

In terms of the political evidence, we have the admission by President Joe Biden on February 7, 2022 during a joint news conference with the newly crowned German Chancellor Olaf Scholz, with Biden issuing categorical threat: “If Russia invades, that means tanks and troops crossing the border of Ukraine again, then there will be no longer a Nord Stream 2,” said Biden, before pausing. “We will bring an end to it.”

Prior to this in late January 2022, Biden’s Under Secretary of State for Political Affairs, Victoria Nuland, issued a similar threat stating, “If Russia invades Ukraine, one way or another, Nord Stream 2 will not move forward.”

Finally, just days after the attack, US Secretary of State Antony Blinken said that the sabotage of Russian Nord Stream gas pipeline was seen in Washington as a “tremendous opportunity” to reduce European energy imports from Russia. Of course, that’s exactly what happened, when after western sanctions against Russian energy imports into Europe took effect, the US and Norway quickly moved in the control the gas import market into northern Europe. Later, in Part 2 of this series, we will also analyse the overarching geostrategic energy play which is now underway, as evidenced by Norway’s renewed interest in the Barents Sea Pipeline gas project as part of Europe’s new Green Deal industrial plan, with its multi-pronged strategy, including long-term ambitions to create a European Hydrogen Energy market, initially involving ‘blue hydrogen’ at first, followed by ‘green hydrogen’ in the long-run. Not surprisingly, the two main partners for this project are Norway and Germany.

The Mission: Nord Stream Sabotage

We believe there is value in being more precise regarding the accusation of that US Navy deep sea divers were used from The Naval Diving and Salvage Training Center (NDSTC) which operates out of the Naval Support Activity base in Panama City (NSA Panama City), often referred to as one of the largest diving facility in the world. Our interest in the subject is not new and those who follow reports on the 21st Century Wire might recall our previous article, “QUI BONO? From 9/11 to Nord Stream – A New Geopolitical Game Changer.” Our curiosity took us to the heart of the US Naval Sea Systems Command (NAVSEA), and its U.S. Navy research and warfare centers (NSWC PCD) in Panama City, including its Navy Experimental Diving Unit (NEDU).

This underwater demolition mission used NATO’s BALTOPS-22 as cover for this act of war, and rapidly our investigation’s attention turned towards the Norwegian Armed Forces and their close ties with NATO Joint Force Command in Norfolk Virginia (NATO JFC-NF), looking into the NATO Wargame Center there, and NATO’s Joint Logistics Support Group (JLSG) based out of Norway – to see if they too played a role in this logistically complex mission. Our findings are more than compelling.

We must first familiarise ourselves with the overarching command structures that this operation was conducted. Let us start with the NAVSEA, currently operates eight Surface Warfare Centers:

1. NSWC Carderock, Maryland.
2. NSWC Corona, California.
3. NSWC Crane, Indiana.
4. NSWC Dahlgren, Virginia.
5. NSWC Indian Head, Maryland.
6. NSWC Panama City, Florida.
7. NSWC Philadelphia, Pennsylvania.
8. NSWC Port Hueneme, California.

NAVSEA has also seven affiliated Program Executive Offices (PEOs) including a PEO for Undersea Warfare Systems which enables the delivery of enhanced combat capability, with improved cybersecurity and resiliency, to all submarine platforms.


Click here to read the complete article
Re: Einstein Paradox: Relativity of Simultaneity

<e501bcba-2d97-417f-8952-e2c01537f141n@googlegroups.com>

  copy mid

https://www.novabbs.com/tech/article-flat.php?id=113961&group=sci.physics.relativity#113961

  copy link   Newsgroups: sci.physics.relativity
X-Received: by 2002:a05:6214:14a6:b0:5ef:4482:a73b with SMTP id bo6-20020a05621414a600b005ef4482a73bmr1448520qvb.2.1682774426064;
Sat, 29 Apr 2023 06:20:26 -0700 (PDT)
X-Received: by 2002:a05:622a:1aa7:b0:3ef:9f86:735d with SMTP id
s39-20020a05622a1aa700b003ef9f86735dmr3057131qtc.13.1682774425856; Sat, 29
Apr 2023 06:20:25 -0700 (PDT)
Path: i2pn2.org!i2pn.org!weretis.net!feeder8.news.weretis.net!proxad.net!feeder1-2.proxad.net!209.85.160.216.MISMATCH!news-out.google.com!nntp.google.com!postnews.google.com!google-groups.googlegroups.com!not-for-mail
Newsgroups: sci.physics.relativity
Date: Sat, 29 Apr 2023 06:20:25 -0700 (PDT)
In-Reply-To: <u2ieve$2rl1n$1@dont-email.me>
Injection-Info: google-groups.googlegroups.com; posting-host=89.206.14.16; posting-account=I3DWzAoAAACOmZUdDcZ-C0PqAZGVsbW0
NNTP-Posting-Host: 89.206.14.16
References: <887d51c1-c60c-4d24-993e-7941d85c4311n@googlegroups.com>
<d2db6956-eba1-4a39-9a09-2a5fbaee5fcdn@googlegroups.com> <u2gpgn$2gg8j$1@dont-email.me>
<0ec41357-3ba8-4f93-9979-d8d0036fa660n@googlegroups.com> <u2ieve$2rl1n$1@dont-email.me>
User-Agent: G2/1.0
MIME-Version: 1.0
Message-ID: <e501bcba-2d97-417f-8952-e2c01537f141n@googlegroups.com>
Subject: Re: Einstein Paradox: Relativity of Simultaneity
From: maluwozn...@gmail.com (Maciej Wozniak)
Injection-Date: Sat, 29 Apr 2023 13:20:26 +0000
Content-Type: text/plain; charset="UTF-8"
Content-Transfer-Encoding: quoted-printable
 by: Maciej Wozniak - Sat, 29 Apr 2023 13:20 UTC

On Saturday, 29 April 2023 at 08:54:41 UTC+2, Volney wrote:
> On 4/28/2023 10:56 PM, gehan.am...@gmail.com wrote:
> > On Friday, April 28, 2023 at 8:42:18 PM UTC+5, Volney wrote:
> >> On 4/28/2023 3:43 AM, JanPB wrote:
> >>> On Tuesday, April 25, 2023 at 2:23:08 PM UTC-7, Jack Liu wrote:
> >>>> Einstein's theory of simultaneity is nothing less than a paradox. I would name it Einstein's Paradox. This should be the biggest paradox in physics in the 20th century. (chapter 9 in "absolute time")
> >>>
> >>> Simultaneity is not essential for SR, it's merely a convenience for
> >>> both derivation and use. But not for the theory as such.
> >> Also, relativity of simultaneity doesn't depend on length
> >> contraction/time dilation in SR in all cases.
> >>
> >> Consider Einstein's train of length L, but is STATIONARY (stopped).
> >> Observer A is on the embankment at the midpoint of the train. Observer B
> >> is on the embankment at the front of the train. Observer C is on the
> >> embankment at the rear.
> >>
> >> Observer A sees simultaneous lightning strikes at the front and rear of
> >> the train. A says the strikes are simultaneous, as the light from each
> >> reaches her at the same time. A observes the strikes L/2c after they
> >> actually happen.
> >>
> >> What does B see? B sees the strike on the front and time L/c later sees
> >> the strike on the rear.
> >>
> >> What does C see? C sees the strike on the rear and time L/c later sees
> >> the strike on the front.
> >>
> >> No SR, no GR, no length contraction or time dilation, nothing but a
> >> finite speed of light. But the three observers all disagree on the
> >> simultaneity of the lightning strikes.
> >
> > This is because the person closer to the lightning strikes sees it first.
> The point is the three observers all disagree which strike happened
> first, with no SR or GR involved whatsoever.

Not quite. The point is that a poor idiot has
fabricated it all.

Re: Einstein Paradox: Relativity of Simultaneity

<8784be74-eef0-4760-9482-026937e7afd2n@googlegroups.com>

  copy mid

https://www.novabbs.com/tech/article-flat.php?id=113967&group=sci.physics.relativity#113967

  copy link   Newsgroups: sci.physics.relativity
X-Received: by 2002:a05:6214:8c8:b0:5ef:4a35:d1e5 with SMTP id da8-20020a05621408c800b005ef4a35d1e5mr1274960qvb.3.1682777458671;
Sat, 29 Apr 2023 07:10:58 -0700 (PDT)
X-Received: by 2002:a05:622a:1a21:b0:3f0:ad19:fa11 with SMTP id
f33-20020a05622a1a2100b003f0ad19fa11mr2947849qtb.11.1682777458284; Sat, 29
Apr 2023 07:10:58 -0700 (PDT)
Path: i2pn2.org!i2pn.org!diablo1.usenet.blueworldhosting.com!usenet.blueworldhosting.com!diablo2.usenet.blueworldhosting.com!peer02.iad!feed-me.highwinds-media.com!news.highwinds-media.com!news-out.google.com!nntp.google.com!postnews.google.com!google-groups.googlegroups.com!not-for-mail
Newsgroups: sci.physics.relativity
Date: Sat, 29 Apr 2023 07:10:58 -0700 (PDT)
In-Reply-To: <80ac7920-ea80-42f4-adbd-1a14574255aen@googlegroups.com>
Injection-Info: google-groups.googlegroups.com; posting-host=191.125.29.18; posting-account=KA67VQoAAAABNtRUVf2Wh-jHtkEfmXxT
NNTP-Posting-Host: 191.125.29.18
References: <887d51c1-c60c-4d24-993e-7941d85c4311n@googlegroups.com>
<kb3plpFa9hkU1@mid.individual.net> <46ffebc6-83dd-4725-bc2f-ac6ebd42a78bn@googlegroups.com>
<80ac7920-ea80-42f4-adbd-1a14574255aen@googlegroups.com>
User-Agent: G2/1.0
MIME-Version: 1.0
Message-ID: <8784be74-eef0-4760-9482-026937e7afd2n@googlegroups.com>
Subject: Re: Einstein Paradox: Relativity of Simultaneity
From: mri...@ing.puc.cl (Paparios)
Injection-Date: Sat, 29 Apr 2023 14:10:58 +0000
Content-Type: text/plain; charset="UTF-8"
Content-Transfer-Encoding: quoted-printable
X-Received-Bytes: 7231
 by: Paparios - Sat, 29 Apr 2023 14:10 UTC

El sábado, 29 de abril de 2023 a las 5:56:07 UTC-4, Jack Liu escribió:
> On Saturday, April 29, 2023 at 2:50:50 AM UTC-5, Prokaryotic Capase Homolog wrote:
> > On Saturday, April 29, 2023 at 12:53:01 AM UTC-5, Sylvia Else wrote:
> > > On 26-Apr-23 7:23 am, Jack Liu wrote:
> > > > Einstein's theory of simultaneity is nothing less than a paradox. I would name it Einstein's Paradox. This should be the biggest paradox in physics in the 20th century. (chapter 9 in "absolute time")
> > > >
> > > > How did Einstein analyze simultaneity relativity? He first assumed that two lightning bolts hit the rails at both ends of the railway "simultaneously", and then he analyzed whether the lightning hit the rails "simultaneously" in the perspective of two observers located at that very moment in the center of the rail.
> > > > The focus is not on whether two observers experience different "simultaneity". The point is that Einstein's reasoning was inconsistent. His conclusion directly denies his premise.
> > > > According to Einstein's conclusion, simultaneity is relative, so the "lightning strikes both ends of the rail simultaneously" in his premise must be conditional. He must specify which observer the "simultaneity" in the premise is relative to.
> > > >
> > > > His Conclusion is against his premise !!!
> > > >
> > > > Again, Einstein's theory of simultaneity is nothing less than a paradox. I would name it Einstein's Paradox. This should be the biggest paradox in physics in the 20th century. (chapter 9 in "absolute time")
> > > >
> > > > for more detail : https://www.amazon.com/Absolute-Time-Relativity-Jack-Liu/dp/B0BQ9JB4RQ
> > > >
> > > I don't see how expect to achieve anything by just making stuff up.
> > > Einstein did not mention lightning in the paper that introduced special
> > > relativity to the world.
> > The famous thought experiment was found in his popular work,
> > "Relativity: The Special and General Theory". In this short book,
> > Einstein translated the formal presentation of his paper into
> > terms more easily grasped by a wide audience.
> >
> > To tell the truth, I was bothered by the thought experiment the
> > first time that I encountered it more than 50 or so years ago.
> > I struggled with the question,
> >
> > | What happens if we try to follow what the primed observer sees,
> > | with the primed frame stationary and the unprimed frame moving?
> > | Wouldn't we witness the light pulses reaching the primed
> > | observer simultaneously, and reaching the unprimed observer at
> > | different times?
> > |
> > | How can a simple shift in viewpoint reverse the results? Is
> > | this possibly a paradox that invalidates the gedanken?
> >
> > It took me years before I worked out the answer.
> >
> > 1) The proper length of the train is LONGER than the proper
> > distance between the lightning strikes.
> > 2) In the frame of the embankment, the moving train is Lorentz-
> > contracted so that its length is the same as the distance
> > between the lightning bolts, which of course when measured in
> > the frame of the embankment is the proper distance.
> > 3) In the frame of the train, the distance between the lightning
> > strikes is Lorentz-contracted so that it is less than the
> > length of the train, which of course when measured in the
> > frame of the train is its proper length.
> >
> > Here is an animation. If you blink at the wrong time, you can
> > miss important events, so be prepared to have to watch it several
> > times before you see everything.
> > https://en.wikipedia.org/wiki/File:Train_and_Embankment_Thought_Experiment_And_Its_Inverse.gif
> How would you work out a answer? that is PARADOX: any conclusion will be against its premise.
>
> In the premise he said two lightning hit road with "simultaneity", the conclusion is two lightning hit road without simultaneity". That is biggest Paradox in Relativity, funnier than Twin Paradox.

Nonsense. First, there is no paradox in this train and embankment thought experiment.
Secondly, you do not need to know anything about SR to derive it.

The explanation is quite simple:

1) You have a very long train (say 120000 km long), with a static observer M' located at the center of the train (ie at 60000 km from the front A and the back B). The train is moving at a very large speed (say 0.8c, or 240000 Km/sec), from left to right.
2) In the embankment you have another observer M which also is static at point O (located at 600000 km from A and B.
3) Just when M' and M are colocated (ie observer M' is passing by observer M) lightning strikes hit the train tracks at points A and B simultaneously at time t=0 (we do not need to measure or verify that, it is an assumption).
4) Observer M, being at point O in the middle, will receive the light from both lightning strikes at the same time (ie simultaneously). Those signals will arrive at M at t=0.2 seconds.
5) When the light from strikes arrives to observer M (at t=0.2 seconds) M' is no longer colocated with M. In fact, M' has moved (with the train) 48000 km to the right of M. Therefore M' will encounter the light coming from point B, before that light arrives to observer M. Also, M' will receive the light coming from point A after that light arrives to M.
6) Therefore, while M saw both lights to occur simultaneously, M' saw first the light from B and then the light from A (ie the light were not simultaneous).

See https://www.marxists.org/reference/archive/einstein/works/1910s/relative/ch09.htm for Einstein's text

Re: Einstein Paradox: Relativity of Simultaneity

<a7d6c2bd-497c-4d3f-b8db-9f98d9828c78n@googlegroups.com>

  copy mid

https://www.novabbs.com/tech/article-flat.php?id=113971&group=sci.physics.relativity#113971

  copy link   Newsgroups: sci.physics.relativity
X-Received: by 2002:ac8:7d41:0:b0:3e8:f79d:bdfa with SMTP id h1-20020ac87d41000000b003e8f79dbdfamr3038465qtb.0.1682777945606;
Sat, 29 Apr 2023 07:19:05 -0700 (PDT)
X-Received: by 2002:a05:622a:289:b0:3ef:4b16:3f38 with SMTP id
z9-20020a05622a028900b003ef4b163f38mr2834160qtw.13.1682777945336; Sat, 29 Apr
2023 07:19:05 -0700 (PDT)
Path: i2pn2.org!i2pn.org!weretis.net!feeder8.news.weretis.net!proxad.net!feeder1-2.proxad.net!209.85.160.216.MISMATCH!news-out.google.com!nntp.google.com!postnews.google.com!google-groups.googlegroups.com!not-for-mail
Newsgroups: sci.physics.relativity
Date: Sat, 29 Apr 2023 07:19:05 -0700 (PDT)
In-Reply-To: <kb4gsbFdn9fU1@mid.individual.net>
Injection-Info: google-groups.googlegroups.com; posting-host=98.197.62.158; posting-account=8fhuCAkAAADWKcWwPpTEbux5C4yu40ip
NNTP-Posting-Host: 98.197.62.158
References: <887d51c1-c60c-4d24-993e-7941d85c4311n@googlegroups.com>
<kb3plpFa9hkU1@mid.individual.net> <a7886d6c-f90c-415c-a7c2-0a52a5a67850n@googlegroups.com>
<kb4gsbFdn9fU1@mid.individual.net>
User-Agent: G2/1.0
MIME-Version: 1.0
Message-ID: <a7d6c2bd-497c-4d3f-b8db-9f98d9828c78n@googlegroups.com>
Subject: Re: Einstein Paradox: Relativity of Simultaneity
From: liu...@gmail.com (Jack Liu)
Injection-Date: Sat, 29 Apr 2023 14:19:05 +0000
Content-Type: text/plain; charset="UTF-8"
Content-Transfer-Encoding: quoted-printable
 by: Jack Liu - Sat, 29 Apr 2023 14:19 UTC

On Saturday, April 29, 2023 at 7:29:03 AM UTC-5, Sylvia Else wrote:
> On 29-Apr-23 8:03 pm, Jack Liu wrote:
> > On Saturday, April 29, 2023 at 12:53:01 AM UTC-5, Sylvia Else wrote:
> >> On 26-Apr-23 7:23 am, Jack Liu wrote:
> >>> Einstein's theory of simultaneity is nothing less than a paradox. I would name it Einstein's Paradox. This should be the biggest paradox in physics in the 20th century. (chapter 9 in "absolute time")
> >>>
> >>> How did Einstein analyze simultaneity relativity? He first assumed that two lightning bolts hit the rails at both ends of the railway "simultaneously", and then he analyzed whether the lightning hit the rails "simultaneously" in the perspective of two observers located at that very moment in the center of the rail.
> >>> The focus is not on whether two observers experience different "simultaneity". The point is that Einstein's reasoning was inconsistent. His conclusion directly denies his premise.
> >>> According to Einstein's conclusion, simultaneity is relative, so the "lightning strikes both ends of the rail simultaneously" in his premise must be conditional. He must specify which observer the "simultaneity" in the premise is relative to.
> >>>
> >>> His Conclusion is against his premise !!!
> >>>
> >>> Again, Einstein's theory of simultaneity is nothing less than a paradox. I would name it Einstein's Paradox. This should be the biggest paradox in physics in the 20th century. (chapter 9 in "absolute time")
> >>>
> >>> for more detail : https://www.amazon.com/Absolute-Time-Relativity-Jack-Liu/dp/B0BQ9JB4RQ
> >>>
> >> I don't see how expect to achieve anything by just making stuff up.
> >> Einstein did not mention lightning in the paper that introduced special
> >> relativity to the world.
> >>
> >> Sylvia.
> >
> > To Sylvia.
> >
> > I don't expect you agree with me for anything. But I agree with you for the following 2 points :
> >
> > 1. that I am the one making stuff up, I am the one to name it Einstein Paradox.
> > 2. that you said Einstein did not mention lightning in the paper that introduced special relativity to the world, which means you don't care it is Paradox since Einstein did not mention lightning in the paper that introduced special relativity.
> >
> > That is good enough. Seems we have finally some agreement.
> >
> > Jack
> >
> No, it means that you should not invent straw men.
>
> Sylvia.

The Scarecrow was created in 1905. I expounded that the relativity of simultaneity is a paradox, but you said that the relativity of simultaneity is not the basis of the theory of relativity. That's how you defend the your straw man. Ha ha.

Re: Einstein Paradox: Relativity of Simultaneity

<8794d277-8d5e-4870-b3e3-2b836c207acen@googlegroups.com>

  copy mid

https://www.novabbs.com/tech/article-flat.php?id=113974&group=sci.physics.relativity#113974

  copy link   Newsgroups: sci.physics.relativity
X-Received: by 2002:a05:622a:56:b0:3f1:fb02:8331 with SMTP id y22-20020a05622a005600b003f1fb028331mr2915938qtw.9.1682778837405;
Sat, 29 Apr 2023 07:33:57 -0700 (PDT)
X-Received: by 2002:ac8:7f03:0:b0:3bf:b9d9:6759 with SMTP id
f3-20020ac87f03000000b003bfb9d96759mr2824400qtk.8.1682778837157; Sat, 29 Apr
2023 07:33:57 -0700 (PDT)
Path: i2pn2.org!i2pn.org!diablo1.usenet.blueworldhosting.com!usenet.blueworldhosting.com!diablo2.usenet.blueworldhosting.com!peer02.iad!feed-me.highwinds-media.com!news.highwinds-media.com!news-out.google.com!nntp.google.com!postnews.google.com!google-groups.googlegroups.com!not-for-mail
Newsgroups: sci.physics.relativity
Date: Sat, 29 Apr 2023 07:33:56 -0700 (PDT)
In-Reply-To: <dJUEdrNPwR_D5ff0989WYMeINQE@jntp>
Injection-Info: google-groups.googlegroups.com; posting-host=98.197.62.158; posting-account=8fhuCAkAAADWKcWwPpTEbux5C4yu40ip
NNTP-Posting-Host: 98.197.62.158
References: <887d51c1-c60c-4d24-993e-7941d85c4311n@googlegroups.com>
<d2db6956-eba1-4a39-9a09-2a5fbaee5fcdn@googlegroups.com> <u2gpgn$2gg8j$1@dont-email.me>
<5cdada26-5b79-4620-8ac1-7602bf5442c5n@googlegroups.com> <c87d1b2f-9e7c-4411-81cd-20a2f271c8c4n@googlegroups.com>
<0747548f-bc47-46d7-b0c2-68d09c3a7e9an@googlegroups.com> <e51ef3fa-5e52-48e6-bf4a-3f8499f876adn@googlegroups.com>
<_5CJAa4qjAloBDuFOEVK0NIA2m0@jntp> <35fc3de8-b325-4be7-804d-1e3364fe1dd7n@googlegroups.com>
<dJUEdrNPwR_D5ff0989WYMeINQE@jntp>
User-Agent: G2/1.0
MIME-Version: 1.0
Message-ID: <8794d277-8d5e-4870-b3e3-2b836c207acen@googlegroups.com>
Subject: Re: Einstein Paradox: Relativity of Simultaneity
From: liu...@gmail.com (Jack Liu)
Injection-Date: Sat, 29 Apr 2023 14:33:57 +0000
Content-Type: text/plain; charset="UTF-8"
Content-Transfer-Encoding: quoted-printable
X-Received-Bytes: 4056
 by: Jack Liu - Sat, 29 Apr 2023 14:33 UTC

On Saturday, April 29, 2023 at 7:01:40 AM UTC-5, Richard Hachel wrote:
> Le 29/04/2023 à 02:38, Jack Liu a écrit :
> >
> > Dear R. H.
> >
> > Thank you for reading my book.
> >
> > That is Lorentz Factor I derive for inbound motion Lorentz Transformation.
> >
> > Einstein consider only outbound moving body and derive a Lorentz factor > 0,
> > which indicate time dilation. That is only half of the whole picture.
> > I apply same logic of Einstein to apply to inbound moving body, and derive to
> > above new Lorentz factor which is <1, which indicate time contraction.
> >
> > I try to demonstrate that, once SP continue to develop another half, it will
> > contradict to itself.
> >
> I don't understand your use of the "+" sign in your equation.
>
> The "1/sqrt(1-v²/c²)" factor proposed by Henri Poincaré (the greatest
> mathematician in the history of mankind and the only one who was able to
> control the entirety of all the science of his time ( physics,
> mathematics, philosophy) which would no longer be possible today as the
> fields have become vast) is correct.
>
> You must ask:
> Vo=Vr/sqrt(1+Vr²/c²)
>
> and Vr=Vo/sqrt(1-Vo²/c²)
>
> These are the two reciprocal equations.
>
> So we have g=1/sqrt(1-Vo²/c²)
>
> or g=sqrt(1+Vr²/c²)
>
> But g=1/sqrt(1+v²/c²) doesn't seem to be of interest.
>
> R.H.

*****************************************************************************************

Dear R.H.

1/sqrt(1-v²/c²) is the factor for two Coordinate Systems departing from each other, which Einstein and Poincaré had considered.
1/sqrt(1+v²/c²) is the factor for two Coordinate Systems approaching each other, which Einstein and Poincaré had not considered.

1/sqrt(1+v²/c²) can be derived mathematically like1/sqrt(1-v²/c²). My book derives them using two different methods, although my derivations are limited to the high school math level.

See Chapter Seven for the derivation process. https://www.amazon.com/Absolute-Time-Relativity-Jack-Liu/dp/B0BQ9JB4RQ

Jack
****************************************************************************************

Re: Einstein Paradox: Relativity of Simultaneity

<2402bd90-f8aa-4f20-8cd7-387865a979bbn@googlegroups.com>

  copy mid

https://www.novabbs.com/tech/article-flat.php?id=113975&group=sci.physics.relativity#113975

  copy link   Newsgroups: sci.physics.relativity
X-Received: by 2002:a05:6214:162c:b0:5ef:63ed:c5d8 with SMTP id e12-20020a056214162c00b005ef63edc5d8mr1469498qvw.3.1682778873755;
Sat, 29 Apr 2023 07:34:33 -0700 (PDT)
X-Received: by 2002:a05:622a:1ba4:b0:3e1:5755:7bbf with SMTP id
bp36-20020a05622a1ba400b003e157557bbfmr3051231qtb.5.1682778873527; Sat, 29
Apr 2023 07:34:33 -0700 (PDT)
Path: i2pn2.org!i2pn.org!weretis.net!feeder6.news.weretis.net!usenet.blueworldhosting.com!diablo2.usenet.blueworldhosting.com!peer02.iad!feed-me.highwinds-media.com!news.highwinds-media.com!news-out.google.com!nntp.google.com!postnews.google.com!google-groups.googlegroups.com!not-for-mail
Newsgroups: sci.physics.relativity
Date: Sat, 29 Apr 2023 07:34:33 -0700 (PDT)
In-Reply-To: <a7d6c2bd-497c-4d3f-b8db-9f98d9828c78n@googlegroups.com>
Injection-Info: google-groups.googlegroups.com; posting-host=2601:646:100:e6a0:75b1:5ee1:ee8a:551c;
posting-account=AZtzIAoAAABqtlvuXL6ZASWM0fV9f6PZ
NNTP-Posting-Host: 2601:646:100:e6a0:75b1:5ee1:ee8a:551c
References: <887d51c1-c60c-4d24-993e-7941d85c4311n@googlegroups.com>
<kb3plpFa9hkU1@mid.individual.net> <a7886d6c-f90c-415c-a7c2-0a52a5a67850n@googlegroups.com>
<kb4gsbFdn9fU1@mid.individual.net> <a7d6c2bd-497c-4d3f-b8db-9f98d9828c78n@googlegroups.com>
User-Agent: G2/1.0
MIME-Version: 1.0
Message-ID: <2402bd90-f8aa-4f20-8cd7-387865a979bbn@googlegroups.com>
Subject: Re: Einstein Paradox: Relativity of Simultaneity
From: l.c.cros...@hotmail.com (Laurence Clark Crossen)
Injection-Date: Sat, 29 Apr 2023 14:34:33 +0000
Content-Type: text/plain; charset="UTF-8"
Content-Transfer-Encoding: quoted-printable
X-Received-Bytes: 4630
 by: Laurence Clark Cross - Sat, 29 Apr 2023 14:34 UTC

On Saturday, April 29, 2023 at 7:19:06 AM UTC-7, Jack Liu wrote:
> On Saturday, April 29, 2023 at 7:29:03 AM UTC-5, Sylvia Else wrote:
> > On 29-Apr-23 8:03 pm, Jack Liu wrote:
> > > On Saturday, April 29, 2023 at 12:53:01 AM UTC-5, Sylvia Else wrote:
> > >> On 26-Apr-23 7:23 am, Jack Liu wrote:
> > >>> Einstein's theory of simultaneity is nothing less than a paradox. I would name it Einstein's Paradox. This should be the biggest paradox in physics in the 20th century. (chapter 9 in "absolute time")
> > >>>
> > >>> How did Einstein analyze simultaneity relativity? He first assumed that two lightning bolts hit the rails at both ends of the railway "simultaneously", and then he analyzed whether the lightning hit the rails "simultaneously" in the perspective of two observers located at that very moment in the center of the rail.
> > >>> The focus is not on whether two observers experience different "simultaneity". The point is that Einstein's reasoning was inconsistent. His conclusion directly denies his premise.
> > >>> According to Einstein's conclusion, simultaneity is relative, so the "lightning strikes both ends of the rail simultaneously" in his premise must be conditional. He must specify which observer the "simultaneity" in the premise is relative to.
> > >>>
> > >>> His Conclusion is against his premise !!!
> > >>>
> > >>> Again, Einstein's theory of simultaneity is nothing less than a paradox. I would name it Einstein's Paradox. This should be the biggest paradox in physics in the 20th century. (chapter 9 in "absolute time")
> > >>>
> > >>> for more detail : https://www.amazon.com/Absolute-Time-Relativity-Jack-Liu/dp/B0BQ9JB4RQ
> > >>>
> > >> I don't see how expect to achieve anything by just making stuff up.
> > >> Einstein did not mention lightning in the paper that introduced special
> > >> relativity to the world.
> > >>
> > >> Sylvia.
> > >
> > > To Sylvia.
> > >
> > > I don't expect you agree with me for anything. But I agree with you for the following 2 points :
> > >
> > > 1. that I am the one making stuff up, I am the one to name it Einstein Paradox.
> > > 2. that you said Einstein did not mention lightning in the paper that introduced special relativity to the world, which means you don't care it is Paradox since Einstein did not mention lightning in the paper that introduced special relativity.
> > >
> > > That is good enough. Seems we have finally some agreement.
> > >
> > > Jack
> > >
> > No, it means that you should not invent straw men.
> >
> > Sylvia.
> The Scarecrow was created in 1905. I expounded that the relativity of simultaneity is a paradox, but you said that the relativity of simultaneity is not the basis of the theory of relativity. That's how you defend the your straw man. Ha ha.
The relativity of simultaneity is a self-contradictory idea, so it is not a paradox. A paradox is something that is apparently contradictory but not really contradictory.

Re: Einstein Paradox: Relativity of Simultaneity

<41458053-e957-4aa2-aeb9-9477df93014fn@googlegroups.com>

  copy mid

https://www.novabbs.com/tech/article-flat.php?id=113977&group=sci.physics.relativity#113977

  copy link   Newsgroups: sci.physics.relativity
X-Received: by 2002:a05:622a:1822:b0:3ef:35e2:addb with SMTP id t34-20020a05622a182200b003ef35e2addbmr3124962qtc.3.1682779429871;
Sat, 29 Apr 2023 07:43:49 -0700 (PDT)
X-Received: by 2002:ad4:5909:0:b0:5ef:5ec2:3ddd with SMTP id
ez9-20020ad45909000000b005ef5ec23dddmr1279740qvb.5.1682779429656; Sat, 29 Apr
2023 07:43:49 -0700 (PDT)
Path: i2pn2.org!i2pn.org!news.neodome.net!feeder1.feed.usenet.farm!feed.usenet.farm!peer02.ams4!peer.am4.highwinds-media.com!peer02.iad!feed-me.highwinds-media.com!news.highwinds-media.com!news-out.google.com!nntp.google.com!postnews.google.com!google-groups.googlegroups.com!not-for-mail
Newsgroups: sci.physics.relativity
Date: Sat, 29 Apr 2023 07:43:49 -0700 (PDT)
In-Reply-To: <b3242fb1-4ce2-40ba-874d-df2cf0385dcen@googlegroups.com>
Injection-Info: google-groups.googlegroups.com; posting-host=98.197.62.158; posting-account=8fhuCAkAAADWKcWwPpTEbux5C4yu40ip
NNTP-Posting-Host: 98.197.62.158
References: <887d51c1-c60c-4d24-993e-7941d85c4311n@googlegroups.com>
<d2db6956-eba1-4a39-9a09-2a5fbaee5fcdn@googlegroups.com> <u2gpgn$2gg8j$1@dont-email.me>
<0ec41357-3ba8-4f93-9979-d8d0036fa660n@googlegroups.com> <u2ieve$2rl1n$1@dont-email.me>
<b3242fb1-4ce2-40ba-874d-df2cf0385dcen@googlegroups.com>
User-Agent: G2/1.0
MIME-Version: 1.0
Message-ID: <41458053-e957-4aa2-aeb9-9477df93014fn@googlegroups.com>
Subject: Re: Einstein Paradox: Relativity of Simultaneity
From: liu...@gmail.com (Jack Liu)
Injection-Date: Sat, 29 Apr 2023 14:43:49 +0000
Content-Type: text/plain; charset="UTF-8"
Content-Transfer-Encoding: base64
X-Received-Bytes: 2895
 by: Jack Liu - Sat, 29 Apr 2023 14:43 UTC

On Saturday, April 29, 2023 at 7:41:10 AM UTC-5, gehan.am...@gmail.com wrote:

> Why does Einstein use this example?

————————————————————————————————————————————————————————
To Gehan
Einstein argued that judgments about time are actually judgments about simultaneity, so he tried to prove that simultaneity is relative.
But What he made was a big joke.
In the proof process, one of his premises is simultaneity. He tried to use the absoluteness of simultaneity to prove the relativity of simultaneity. This is the greatest paradox of contemporary physics. This paradox is exposed and critiqued in detail in Chapter 9 of my book.
https://drive.google.com/file/d/1VfhOL63jvB2Dmn4JCRmOx6S8Dh9nRbdC/view
Jack
——————————————————————————————————————————————————————————


tech / sci.physics.relativity / Re: Einstein Paradox: Relativity of Simultaneity

Pages:1234
server_pubkey.txt

rocksolid light 0.9.81
clearnet tor